Site Loader

Содержание

Векторное произведение векторов площадь треугольника. Свойства векторного произведения

Определение. Векторным произведением вектора а (множимое) на не коллинеарный ему вектор (множитель) называется третий вектор с (произведение), который строится следующим образом:

1) его модуль численно равен площади параллелограмма на рис. 155), построенного на векторах т. е. он равен направление перпендикулярно плоскости упомянутого параллелограмма;

3) при этом направление вектора с выбирается (из двух возможных) так, чтобы векторы с составляли правую систему (§ 110).

Обозначение: или

Дополнение к определению. Если векторы коллинеарны, то фигуре считая ее (условно) параллелограммом, естественно приписать нулевую площадь. Поэтому векторное произведение коллинеарных векторов считается равным нуль-вектору.

Поскольку нуль-вектору можно приписать любое направление, это соглашение не противоречит пунктам 2 и 3 определения.

Замечание 1. В термине «векторное произведение» первое слово указывает на то, что результат действия есть вектор (в противоположность скалярному произведению; ср. § 104, замечание 1).

Пример 1. Найти векторное произведение где основные векторы правой системы координат (рис. 156).

1. Так как длины основных векторов равны единице масштаба, то площадь параллелограмма (квадрата) численно равна единице. Значит, модуль векторного произведения равен единице.

2. Так как перпендикуляр к плоскости есть ось то искомое векторное произведение есть вектор, коллинеарный вектору к; а так как оба они имеют модуль 1, то искомое векторное произведение равно либо k, либо -k.

3. Из этих двух возможных векторов надо выбрать первый, так как векторы к образуют правую систему (а векторы левую).

Пример 2. Найти векторное произведение

Решение. Как в примере 1, заключаем, что вектор равен либо k, либо -k. Но теперь надо выбрать -k, так как векторы образуют правую систему (а векторы левую). Итак,

Пример 3. Векторы имеют длины, соответственно равные 80 и 50 см, и образуют угол 30°. Приняв за единицу длины метр, найти длину векторного произведения а

Решение. Площадь параллелограмма, построенного на векторах равна Длина искомого векторного произведения равна

Пример 4. Найти длину векторного произведения тех же векторов, приняв за единицу длины сантиметр.

Решение. Так как площадь параллелограмма, построенного на векторах равна то длина векторного произведения равна 2000 см, т. е.

Из сравнения примеров 3 и 4 видно, что длина вектора зависит не только от длин сомножителей но также и от выбора единицы длины.

Физический смысл векторного произведения. Из многочисленных физических величин, изображаемых векторным произведением, рассмотрим только момент силы.

Пусть А есть точка приложения силы Моментом силы относителько точки О называется векторное произведение Так как модуль этого векторного произведения численно равен площади параллелограмма (рис. 157), то модуль момента равняется произведению основания на высоту т. е. силе, умноженной на расстояние от точки О до прямой, вдоль которой действует сила.

В механике доказывается, что для равновесия твердого тела необходимо, чтобы равнялась нулю не только сумма векторов , представляющих силы, приложенные к телу, но также и сумма моментов сил. j ;

2) |k |=1, но | i x j | = |i | |J | sin(90°)=1;

3) векторы i , j и k образуют правую тройку (см. рис. 16).

7.2. Свойства векторного произведения

1. При перестановке сомножителей векторное произведение меняет знак, т.е. а хb =(b хa ) (см. рис. 19).

Векторы а хb и b ха коллинеарны, имеют одинаковые модули (площадь параллелограмма остается неизменной), но противоположно направлены (тройки а , b , а хb и a , b , b x a противоположной ориентации). Стало быть a xb = -(b xa ).

2. Векторное произведение обладает сочетательным свойством относительно скалярного множителя, т. е. l (а хb ) = (l а ) х b = а х (l b ).

Пусть l >0. Вектор l (а хb ) перпендикулярен векторам а и b . Вектор ( l а )хb также перпендикулярен векторам а и b (векторы а , l а лежат в одной плоскости). Значит, векторы l (а хb ) и ( l а )хb коллинеарны. Очевидно, что и направления их совпадают. Имеют одинаковую длину:

Поэтому l (a хb )= l а хb . Аналогично доказывается при l

3. Два ненулевых вектора а и b коллинеарны тогда и только тогда, когда их векторное произведение равно нулевому вектору, т. е. а ||b а хb =0 .

В частности, i *i =j *j =k *k =0 .

4. Векторное произведение обладает распределительным свойством:

(a +b ) хс = а хс +b хс .

Примем без доказательства.

7.3. Выражение векторного произведения через координаты

Мы будем использовать таблицу векторного произведения векторов i , j и k :

если направление кратчайшего пути от первого вектора к второму совпадает с направлением стрелки, то произведение равно третьему вектору, если не совпадает — третий вектор берется со знаком «минус».

Пусть заданы два вектора а =а х i +a y j +a z k и b =b x i +b y j +b z k . Найдем векторное произведение этих векторов, перемножая их как многочлены (согласно свойств векторного произведения):



Полученную формулу можно записать еще короче:

так как правая часть равенства (7.1) соответствует разложению определителя третьего порядка по элементам первой строки.Равенство (7.2) легко запоминается.

7.4. Некоторые приложения векторного произведения

Установление коллинеарности векторов

Нахождение площади параллелограмма и треугольника

Согласно определению векторного произведения векторов а и b |а хb | = |а | * |b |sin g , т. е. S пар = |а х b |. И, значит, D S =1/2|а х b |.

Определение момента силы относительно точки

Пусть в точке А приложена сила F =АВ и пусть О — некоторая точка пространства (см. рис. 20).

Из физики известно, что моментом си лы F относительно точки О называется вектор М , который проходит через точку О и:

1) перпендикулярен плоскости, проходящей через точки О, А, В;

2) численно равен произведению силы на плечо

3) образует правую тройку с векторами ОА и A В .

Стало быть, М =ОА х F .

Нахождение линейной скорости вращения

Скорость v точки М твердого тела, вращающегося с угловой скоростью w вокруг неподвижной оси, определяется формулой Эйлера v =w хr , где r =ОМ , где О-некоторая неподвижная точка оси (см. рис. 21).

На данном уроке мы рассмотрим ещё две операции с векторами: векторное произведение векторов и смешанное произведение векторов (сразу ссылка, кому нужно именно оно) . Ничего страшного, так иногда бывает, что для полного счастья, помимо скалярного произведения векторов , требуется ещё и ещё. Такая вот векторная наркомания. Может сложиться впечатление, что мы залезаем в дебри аналитической геометрии. Это не так. В данном разделе высшей математики вообще мало дров, разве что на Буратино хватит. На самом деле материал очень распространенный и простой – вряд ли сложнее, чем то же

скалярное произведение , даже типовых задач поменьше будет. Главное в аналитической геометрии, как многие убедятся или уже убедились, НЕ ОШИБАТЬСЯ В ВЫЧИСЛЕНИЯХ. Повторяйте как заклинание, и будет вам счастье =)

Если векторы сверкают где-то далеко, как молнии на горизонте, не беда, начните с урока Векторы для чайников , чтобы восстановить или вновь приобрести базовые знания о векторах. Более подготовленные читатели могут знакомиться с информацией выборочно, я постарался собрать максимально полную коллекцию примеров, которые часто встречаются в практических работах

Чем вас сразу порадовать? Когда я был маленьким, то умел жонглировать двумя и даже тремя шариками. Ловко получалось. Сейчас жонглировать не придётся вообще, поскольку мы будем рассматривать

только пространственные векторы , а плоские векторы с двумя координатами останутся за бортом. Почему? Такими уж родились данные действия – векторное и смешанное произведение векторов определены и работают в трёхмерном пространстве. Уже проще!

В данной операции, точно так же, как и в скалярном произведении, участвуют два вектора . Пусть это будут нетленные буквы .

Само действие обозначается следующим образом: . Существуют и другие варианты, но я привык обозначать векторное произведение векторов именно так, в квадратных скобках с крестиком.

И сразу вопрос : если в скалярном произведении векторов

участвуют два вектора, и здесь тоже умножаются два вектора, тогда в чём разница ? Явная разница, прежде всего, в РЕЗУЛЬТАТЕ:

Результатом скалярного произведения векторов является ЧИСЛО:

Результатом векторного произведения векторов является ВЕКТОР : , то есть умножаем векторы и получаем снова вектор. Закрытый клуб. Собственно, отсюда и название операции. В различной учебной литературе обозначения тоже могут варьироваться, я буду использовать букву .

Определение векторного произведения

Сначала будет определение с картинкой, затем комментарии.

Определение : Векторным произведением неколлинеарных векторов , взятых в данном порядке

, называется ВЕКТОР , длина которого численно равна площади параллелограмма , построенного на данных векторах; вектор ортогонален векторам , и направлен так, что базис имеет правую ориентацию:

Разбираем определение по косточкам, тут много интересного!

Итак, можно выделить следующие существенные моменты:

1) Исходные векторы , обозначенные красными стрелками, по определению не коллинеарны . Случай коллинеарных векторов будет уместно рассмотреть чуть позже.

2) Векторы взяты в строго определённом порядке : – «а» умножается на «бэ» , а не «бэ» на «а». Результатом умножения векторов является ВЕКТОР , который обозначен синим цветом. Если векторы умножить в обратном порядке, то получим равный по длине и противоположный по направлению вектор (малиновый цвет). То есть, справедливо равенство .

3) Теперь познакомимся с геометрическим смыслом векторного произведения. Это очень важный пункт! ДЛИНА синего вектора (а, значит, и малинового вектора ) численно равна ПЛОЩАДИ параллелограмма, построенного на векторах . На рисунке данный параллелограмм заштрихован чёрным цветом.

Примечание : чертёж является схематическим, и, естественно, номинальная длина векторного произведения не равна площади параллелограмма.

Вспоминаем одну из геометрических формул: площадь параллелограмма равна произведению смежных сторон на синус угла между ними . Поэтому, исходя из вышесказанного, справедлива формула вычисления ДЛИНЫ векторного произведения:

Подчёркиваю, что в формуле речь идёт о ДЛИНЕ вектора, а не о самом векторе . Каков практический смысл? А смысл таков, что в задачах аналитической геометрии площадь параллелограмма часто находят через понятие векторного произведения:

Получим вторую важную формулу. Диагональ параллелограмма (красный пунктир) делит его на два равных треугольника. Следовательно, площадь треугольника, построенного на векторах (красная штриховка), можно найти по формуле:

4) Не менее важный факт состоит в том, что вектор ортогонален векторам , то есть . Разумеется, противоположно направленный вектор (малиновая стрелка) тоже ортогонален исходным векторам .

5) Вектор направлен так, что базис имеет правую ориентацию. На уроке о переходе к новому базису я достаточно подробно рассказал об ориентации плоскости , и сейчас мы разберёмся, что такое ориентация пространства. Объяснять буду на пальцах вашей правой руки . Мысленно совместите указательный палец с вектором и средний палец с вектором . Безымянный палец и мизинец прижмите к ладони. В результате большой палец – векторное произведение будет смотреть вверх. Это и есть правоориентированный базис (на рисунке именно он). Теперь поменяйте векторы (указательный и средний пальцы ) местами, в результате большой палец развернётся, и векторное произведение уже будет смотреть вниз. Это тоже правоориентированный базис. Возможно, у вас возник вопрос: а какой базис имеет левую ориентацию? «Присвойте» тем же пальцам левой руки векторы , и полУчите левый базис и левую ориентацию пространства (в этом случае большой палец расположится по направлению нижнего вектора) . Образно говоря, данные базисы «закручивают» или ориентируют пространство в разные стороны. И это понятие не следует считать чем-то надуманным или абстрактным – так, например, ориентацию пространства меняет самое обычное зеркало, и если «вытащить отражённый объект из зазеркалья», то его в общем случае не удастся совместить с «оригиналом». Кстати, поднесите к зеркалу три пальца и проанализируйте отражение;-)

…как всё-таки хорошо, что вы теперь знаете о право- и левоориентированных базисах, ибо страшнЫ высказывания некоторых лекторов о смене ориентации =)

Векторное произведение коллинеарных векторов

Определение подробно разобрано, осталось выяснить, что происходит, когда векторы коллинеарны. Если векторы коллинеарны, то их можно расположить на одной прямой и наш параллелограмм тоже «складывается» в одну прямую. Площадь такого, как говорят математики, вырожденного параллелограмма равна нулю. Это же следует и из формулы – синус нуля или 180-ти градусов равен нулю, а значит, и площадь нулевая

Таким образом, если , то . Строго говоря, само векторное произведение равно нулевому вектору, но на практике этим часто пренебрегают и пишут, что оно просто равно нулю.

Частный случай – векторное произведение вектора на самого себя:

С помощью векторного произведения можно проверять коллинеарность трёхмерных векторов, и данную задачу среди прочих мы тоже разберём.

Для решения практических примеров может потребоваться тригонометрическая таблица , чтобы находить по ней значения синусов.

Ну что же, разжигаем огонь:

Пример 1

а) Найти длину векторного произведения векторов , если

б) Найти площадь параллелограмма, построенного на векторах , если

Решение : Нет, это не опечатка, исходные данные в пунктах условия я намеренно сделал одинаковыми. Потому что оформление решений будет отличаться!

а) По условию требуется найти длину вектора (векторного произведения). По соответствующей формуле:

Ответ :

Коль скоро спрашивалось о длине, то в ответе указываем размерность – единицы.

б) По условию требуется найти площадь параллелограмма, построенного на векторах . Площадь данного параллелограмма численно равна длине векторного произведения:

Ответ :

Обратите внимание, что в ответе о векторном произведении речи не идёт вообще, нас спрашивали о площади фигуры , соответственно, размерность – квадратные единицы.

Всегда смотрим, ЧТО требуется найти по условию, и, исходя из этого, формулируем чёткий ответ. Может показаться буквоедством, но буквоедов среди преподавателей хватает, и задание с хорошими шансами вернётся на доработку. Хотя это не особо натянутая придирка – если ответ некорректен, то складывается впечатление, что человек не разбирается в простых вещах и/или не вник в суть задания. Этот момент всегда нужно держать на контроле, решая любую задачу по высшей математике, да и по другим предметам тоже.

Куда подевалась большая буковка «эн»? В принципе, её можно было дополнительно прилепить в решение, но в целях сократить запись, я этого не сделал. Надеюсь, всем понятно, что и – это обозначение одного и того же.

Популярный пример для самостоятельного решения:

Пример 2

Найти площадь треугольника, построенного на векторах , если

Формула нахождения площади треугольника через векторное произведение дана в комментариях к определению. Решение и ответ в конце урока.

На практике задача действительно очень распространена, треугольниками вообще могут замучить.

Для решения других задач нам понадобятся:

Свойства векторного произведения векторов

Некоторые свойства векторного произведения мы уже рассмотрели, тем не менее, я их включу в данный список.

Для произвольных векторов и произвольного числа справедливы следующие свойства:

1) В других источниках информации данный пункт обычно не выделяют в свойствах, но он очень важен в практическом плане. Поэтому пусть будет.

2) – свойство тоже разобрано выше, иногда его называют антикоммутативностью . Иными словами, порядок векторов имеет значение.

3) – сочетательные или ассоциативные законы векторного произведения. Константы безпроблемно выносятся за пределы векторного произведения. Действительно, чего им там делать?

4) – распределительные или дистрибутивные законы векторного произведения. С раскрытием скобок тоже нет проблем.

В качестве демонстрации рассмотрим коротенький пример:

Пример 3

Найти , если

Решение: По условию снова требуется найти длину векторного произведения. Распишем нашу миниатюру:

(1) Согласно ассоциативным законам, выносим константы за переделы векторного произведения.

(2) Выносим константу за пределы модуля, при этом модуль «съедает» знак «минус». Длина же не может быть отрицательной.

(3) Дальнейшее понятно.

Ответ :

Пора подбросить дров в огонь:

Пример 4

Вычислить площадь треугольника, построенного на векторах , если

Решение : Площадь треугольника найдём по формуле . Загвоздка состоит в том, что векторы «цэ» и «дэ» сами представлены в виде сумм векторов. Алгоритм здесь стандартен и чем-то напоминает примеры № 3 и 4 урока Скалярное произведение векторов . Решение для ясности разобьём на три этапа:

1) На первом шаге выразим векторное произведение через векторное произведение , по сути, выразим вектор через вектор . О длинах пока ни слова!

(1) Подставляем выражения векторов .

(2) Используя дистрибутивные законы, раскрываем скобки по правилу умножения многочленов.

(3) Используя ассоциативные законы, выносим все константы за пределы векторных произведений. При маломальском опыте действия 2 и 3 можно выполнять одновременно.

(4) Первое и последнее слагаемое равно нулю (нулевому вектору) благодаря приятному свойству . Во втором слагаемом используем свойство антикоммутативности векторного произведения:

(5) Приводим подобные слагаемые.

В результате вектор оказался выражен через вектор, чего и требовалось достичь:

2) На втором шаге найдем длину нужного нам векторного произведения. Данное действие напоминает Пример 3:

3) Найдём площадь искомого треугольника:

Этапы 2-3 решения можно было оформить и одной строкой.

Ответ :

Рассмотренная задача достаточно распространена в контрольных работах, вот пример для самостоятельного решения:

Пример 5

Найти , если

Краткое решение и ответ в конце урока. Посмотрим, насколько вы были внимательны при изучении предыдущих примеров;-)

Векторное произведение векторов в координатах , заданных в ортонормированном базисе , выражается формулой :

Формула и правда простецкая: в верхнюю строку определителя записываем координатные векторы, во вторую и третью строки «укладываем» координаты векторов , причём укладываем в строгом порядке – сначала координаты вектора «вэ», затем координаты вектора «дубль-вэ». Если векторы нужно умножить в другом порядке, то и строки следует поменять местами:

Пример 10

Проверить, будут ли коллинеарны следующие векторы пространства:
а)
б)

Решение : Проверка основана на одном из утверждений данного урока: если векторы коллинеарны, то их векторное произведение равно нулю (нулевому вектору): .

а) Найдём векторное произведение:

Таким образом, векторы не коллинеарны.

б) Найдём векторное произведение:

Ответ : а) не коллинеарны, б)

Вот, пожалуй, и все основные сведения о векторном произведении векторов.

Данный раздел будет не очень большим, так как задач, где используется смешанное произведение векторов, немного. Фактически всё будет упираться в определение, геометрический смысл и пару рабочих формул.

Смешанное произведение векторов – это произведение трёх векторов :

Вот так вот они выстроились паровозиком и ждут, не дождутся, когда их вычислят.

Сначала опять определение и картинка:

Определение : Смешанным произведением некомпланарных векторов , взятых в данном порядке , называется объём параллелепипеда , построенного на данных векторах, снабжённый знаком «+», если базис правый, и знаком «–», если базис левый.

Выполним рисунок. Невидимые нам линии прочерчены пунктиром:

Погружаемся в определение:

2) Векторы взяты в определённом порядке , то есть перестановка векторов в произведении , как вы догадываетесь, не проходит без последствий.

3) Перед тем, как прокомментировать геометрический смысл, отмечу очевидный факт: смешанное произведение векторов является ЧИСЛОМ : . В учебной литературе оформление может быть несколько другим, я привык обозначать смешанное произведение через , а результат вычислений буквой «пэ».

По определению смешанное произведение – это объем параллелепипеда , построенного на векторах (фигура прочерчена красными векторами и линиями чёрного цвета). То есть, число равно объему данного параллелепипеда.

Примечание : чертёж является схематическим.

4) Не будем заново париться с понятием ориентации базиса и пространства. Смысл заключительной части состоит в том, что к объёму может добавляться знак минус. Простыми словами, смешанное произведение может быть отрицательным: .

Непосредственно из определения следует формула вычисления объема параллелепипеда, построенного на векторах .

В этой статье мы подробно остановимся на понятии векторного произведения двух векторов. Мы дадим необходимые определения, запишем формулу для нахождения координат векторного произведения, перечислим и обоснуем его свойства. После этого остановимся на геометрическом смысле векторного произведения двух векторов и рассмотрим решения различных характерных примеров.

Навигация по странице.

Определение векторного произведения.

Прежде чем дать определение векторного произведения, разберемся с ориентацией упорядоченной тройки векторов в трехмерном пространстве.

Отложим векторы от одной точки. В зависимости от направления вектора тройка может быть правой или левой. Посмотрим с конца вектора на то, как происходит кратчайший поворот от вектора к . Если кратчайший поворот происходит против часовой стрелки, то тройка векторов называется правой , в противном случае – левой .


Теперь возьмем два не коллинеарных вектора и . Отложим от точки А векторы и . Построим некоторый вектор , перпендикулярный одновременно и и . Очевидно, что при построении вектора мы можем поступить двояко, задав ему либо одно направление, либо противоположное (смотрите иллюстрацию).


В зависимости от направления вектора упорядоченная тройка векторов может быть правой или левой.

Так мы вплотную подошли к определению векторного произведения. Оно дается для двух векторов, заданных в прямоугольной системе координат трехмерного пространства.

Определение.

Векторным произведением двух векторов и , заданных в прямоугольной системе координат трехмерного пространства, называется такой вектор , что

Векторное произведение векторов и обозначается как .

Координаты векторного произведения.

Сейчас дадим второе определение векторного произведения, которое позволяет находить его координаты по координатам заданных векторов и.

Определение.

В прямоугольной системе координат трехмерного пространства векторное произведение двух векторов и есть вектор , где — координатные векторы.

Это определение дает нам векторное произведение в координатной форме.

Векторное произведение удобно представлять в виде определителя квадратной матрицы третьего порядка, первая строка которой есть орты , во второй строке находятся координаты вектора , а в третьей – координаты вектора в заданной прямоугольной системе координат:

Если разложить этот определитель по элементам первой строки, то получим равенство из определения векторного произведения в координатах (при необходимости обращайтесь к статье ):

Следует отметить, что координатная форма векторного произведения полностью согласуется с определением, данным в первом пункте этой статьи. Более того, эти два определения векторного произведения эквивалентны. Доказательство этого факта можете посмотреть в книге, указанной в конце статьи.

Свойства векторного произведения.

Так как векторное произведение в координатах представимо в виде определителя матрицы , то на основании легко обосновываются следующие свойства векторного произведения :

Для примера докажем свойство антикоммутативности векторного произведения.

По определению и . Нам известно, что значение определителя матрицы изменяется на противоположное, если переставить местами две строки, поэтому, , что доказывает свойство антикоммутативности векторного произведения.

Векторное произведение – примеры и решения.

В основном встречаются три типа задач.

В задачах первого типа заданы длины двух векторов и угол между ними, а требуется найти длину векторного произведения. В этом случае используется формула .

Пример.

Найдите длину векторного произведения векторов и , если известно .

Решение.

Мы знаем из определения, что длина векторного произведения векторов и равна произведению длин векторов и на синус угла между ними, поэтому, .

Ответ:

.

Задачи второго типа связаны с координатами векторов, в них векторное произведение, его длина или что-либо еще ищется через координаты заданных векторов и .

Здесь возможна масса различных вариантов. К примеру, могут быть заданы не координаты векторов и , а их разложения по координатным векторам вида и , или векторы и могут быть заданы координатами точек их начала и конца.

Рассмотрим характерные примеры.

Пример.

В прямоугольной системе координат заданы два вектора . Найдите их векторное произведение.

Решение.

По второму определению векторное произведение двух векторов в координатах записывается как:

К такому же результату мы бы пришли, если бы векторное произведение записали через определитель

Ответ:

.

Пример.

Найдите длину векторного произведения векторов и , где — орты прямоугольной декартовой системы координат.

Решение.

Сначала найдем координаты векторного произведения в заданной прямоугольной системе координат.

Так как векторы и имеют координаты и соответственно (при необходимости смотрите статью координаты вектора в прямоугольной системе координат), то по второму определению векторного произведения имеем

То есть, векторное произведение имеет координаты в заданной системе координат.

Длину векторного произведения находим как корень квадратный из суммы квадратов его координат (эту формулу длины вектора мы получили в разделе нахождение длины вектора):

Ответ:

.

Пример.

В прямоугольной декартовой системе координат заданы координаты трех точек . Найдите какой-нибудь вектор, перпендикулярный и одновременно.

Решение.

Векторы и имеют координаты и соответственно (смотрите статью нахождение координат вектора через координаты точек). Если найти векторное произведение векторов и , то оно по определению является вектором, перпендикулярным и к и к , то есть, является решением нашей задачи. Найдем его

Ответ:

— один из перпендикулярных векторов.

В задачах третьего типа проверяется навык использования свойств векторного произведения векторов. После применения свойств, применяются соответствующие формулы.

Пример.

Векторы и перпендикулярны и их длины равны соответственно 3 и 4 . Найдите длину векторного произведения .

Решение.

По свойству дистрибутивности векторного произведения мы можем записать

В силу сочетательного свойства вынесем числовые коэффициенты за знак векторных произведений в последнем выражении:

Векторные произведения и равны нулю, так как и , тогда .

Так как векторное произведение антикоммутативно, то .

Итак, с помощью свойств векторного произведения мы пришли к равенству .

По условию векторы и перпендикулярны, то есть угол между ними равен . То есть, у нас есть все данные для нахождения требуемой длины

Ответ:

.

Геометрический смысл векторного произведения.

По определению длина векторного произведения векторов равна . А из курса геометрии средней школы нам известно, что площадь треугольника равна половине произведения длин двух сторон треугольника на синус угла между ними. Следовательно, длина векторного произведения равна удвоенной площади треугольника, имеющего сторонами векторы и , если их отложить от одной точки. Другими словами, длина векторного произведения векторов и равна площади параллелограмма со сторонами и и углом между ними, равным . В этом состоит геометрический смысл векторного произведения.

Единичный вектор — это вектор , абсолютная величина (модуль) которого равен единице. Для обозначения единичного вектора мы будем использовать нижний индекс е. Так, если задан вектор а , то его единичным вектором будет вектор а е. Этот единичный вектор направлен туда же, куда направлен и сам вектор а , и его модуль равен единице, то есть а е = 1.

Очевидно, а = а·а е (а модуль вектора а) . Это следует из правила, по которому выполняется операция умножения скаляра на вектор .

Единичные векторы часто связывают с координатными осями системы координат (в частности, с осями декартовой системы координат). Направления этих векторов совпадают с направлениями соответствующих осей, а их начала часто совмещают с началом системы координат.

Напомню, что декартовой системой координат в пространстве традиционно называется тройка взаимно перпендикулярных осей, пересекающихся в точке, которая называется началом координат. Координатные оси обычно обозначают буквами X , Y , Z и называют соответственно осью абсцисс, осью ординат и осью аппликат. Сам Декарт пользовался только одной осью, на которой откладывались абсциссы. Заслуга использования системы осей принадлежит его ученикам. Поэтому фраза декартова система координат исторически ошибочна. Лучше говорить прямоугольная система координат или ортогональная система координат . Тем не менее, изменять традиции мы не станем и в дальнейшем будем считать, что декартова и прямоугольная (ортогональная) системы координат — это одно и то же.

Единичный вектор , направленный вдоль оси Х, обозначается i , единичный вектор , направленный вдоль оси Y , обозначается j , а единичный вектор , направленный вдоль оси Z, обозначается k . Векторы i , j , k называются ортами (рис. 12, слева), они имеют единичные модули, то есть
i = 1, j = 1, k = 1.

Оси и орты прямоугольной системы координат в некоторых случаях имеют другие названия и обозначения. Так, ось абсцисс X может называться касательной осью, а ее орт обозначается τ (греческая строчная буква тау), ось ординат — осью нормали, ее орт обозначается n , ось аппликат — осью бинормали, ее орт обозначается b . Зачем менять названия, если суть остается той же?

Дело в том, что, например, в механике при изучении движения тел прямоугольная система координат используется очень часто. Так вот, если сама система координат неподвижна, а изменение координат движущегося объекта отслеживается в этой неподвижной системе, то обычно оси обозначают X, Y, Z, а их орты соответственно i , j , k .

Но нередко, когда объект движется по какой-то криволинейной траектории (например, по окружности) бывает удобнее рассматривать механические процессы в системе координат, движущейся с этим объектом. Именно для такой движущейся системы координат и используются другие названия осей и их ортов. Просто так принято. В этом случае ось X направляют по касательной к траектории в той ее точке, в которой в данный момент этот объект находится. И тогда эту ось называют уже не осью X, а касательной осью, а ее орт обозначают уже не i , а τ . Ось Y направляют по радиусу кривизны траектории (в случае движения по окружности — к центру окружности). А поскольку радиус перпендикулярен касательной, то ось называют осью нормали (перпендикуляр и нормаль — это одно и то же). Орт этой оси обозначают уже не j , а n . Третья ось (бывшая Z) перпендикулярна двум предыдущим. Это — бинормаль с ортом b (рис. 12, справа). Кстати, в этом случае такую прямоугольную систему координат часто называют «естественной» или натуральной.

Векторное произведение — определения, теоремы и примеры

Содержание:

  1. Определение
  2. Смешанное произведение 
  3. Свойства смешанного произведения
  4. Двойное векторное произведение

Определение

Упорядоченная тройка некомпланарных векторов называется правой, если (после совмещения их начал) кратчайший поворот от вектора к вектору виден из конца вектора совершающимся против часовой стрелки. В противном случае упорядоченная тройка некомпланарных векторов называется левой.

Определение

Векторным произведением неколлинеарных векторов и называется вектор , такой, что

В случае, когда сомножители коллинеарны (в том числе, когда хотя бы один из сомножителей есть нулевой вектор), векторное произведение считается равным нулевому вектору.

Векторное произведение векторов и обозначается как . Из определения 2.4.2 следует, что 1°. есть площадь параллелограмма, построенного на векторах и .

2°. Для коллинеарности ненулевых векторов и необходимо и достаточно, чтобы их векторное произведение было равно нулевому вектору. Свойства векторного произведения

1°.(антикоммутативность, следует из определения 2.4.2 и нечетности функции ).

2°. (следует из определения векторного про-

изведения и того факта, что векторы и ортотональны одной и той же плоскости при неколлинеарных и и )

3°. (дистрибутивность).

Для доказательства дистрибутивности векторного произведения воспользуемся следующими вспомогательными утверждениями.

По этой ссылке вы найдёте полный курс лекций по высшей математике:

Лемма. Пусть даны два вектора и , начала которых находятся в общей точке на оси с базисом . Тогда результат поворота суммы векторов и на угол вокруг оси равен сумме результатов поворота каждого из этих векторов вокруг осина угол .

Утверждение леммы 2.4.1 будем обозначать как

Рис. 2.4.1 Его справедливость ясна из рис. 2.4.1. Лемма 2.4.2. Если и ,то вектор равен результату поворота проекции вектора на плоскость, перпендикулярную вектору , вокруг вектора на угол по часовой стрелке.

Возможно вам будут полезны данные страницы:

Доказательство.

Проведем две плоскости, одна из которых проходит через точку О — общее начало векторов и — перпендикулярно , а вторая проходит через векторы и .

Ортогональная проекция вектора на плоскость, перпендикулярную , будет лежать на линии пересечения построенных плоскостей, и тогда из определения векторного произведения следует (рис. 2.4.2) поскольку .

Следовательно, в рассматриваемом случае

обозначает ортогональное проектирование вектора на плоскость, перпендикулярную вектору .

Рис. 2.4.2 Лемма доказана.

Докажем теперь дистрибутивность векторного произведения.

Доказательство свойства 3°.

Если , то свойство 3° очевидно. Пусть , тогда в силу утверждений лемм 2.4.1, 2.4.2 и свойства 1.1° из § 2.1 следует

Свойство доказано.

Смешанное произведение 

Теорема Абсолютная величина смешанного произведения векторов равна объему параллелепипеда, построенного на векторах . При этом если тройка векторов некомпланарная и правая, то их смешанное произведение положительно, а если тройка левая, то — отрицательно.

Доказательство.

Если коллинеарен ), то утверждение теоремы очевидно.

Пусть неколлинеарен тогда по определению скалярного произведения

где есть площадь параллелограмма, постро-енного на векторах ,а — высота параллелепипеда с основанием, откуда (см. рис. 2.6.1)

Наконец,

что и позволяет сделать заключение о знаке смешанного произведения.

Рис. 2.6.1 Теорема доказана.

Свойства смешанного произведения

Для смешанного произведения справедливы тождества:

1°.

2°.

3°.

справедливость которых следует из определения смешанного произведения и теоремы 2.6.1.

Отметим, наконец, что смешанное произведение равно нулю, если среди сомножителей имеется хотя бы одна пара коллинеарных векторов.

Двойное векторное произведение

Определение 2.8.1. Двойным векторным произведением векторов и называется вектор.

Для решения ряда задач оказывается полезной и чуть ниже примеры с решением.

Теорема 2.8.1. Имеет место равенство

Доказательство.

Заметим, что, если векторы попарно ортогональны, то

доказываемое равенство очевидно, поэтому далее будем предполагать, что числа и не равны нулю одновременно. Обозначим По определению векторного произведения векторортогонален как вектору , так и .

1°. По свойствам смешанного произведения условие означает, что тройка векторов компланарная и, в силу леммы 1.4.1, где инекоторые числа.

2°. Из условия следует, что

3°. Рассмотрим теперь вектор , удовлетворяющий следующему набору условий:

а) (также как и вектор ) принадлежит плоскости, проходящей через векторы ;

б) (См. рис. 2.8.1.)

Найдем теперь выражение для смешанного произведения вида. С одной стороны, по свойствам

смешанного произведения и в силу , имеем

Рисунок

2.8.1.

С другой стороны, вектор сонаправлен с , то есть

такое, что . Поэтому

Значение найдем из соотношений

поскольку угол между и прямой. Значит

Приравнивая выражения для , получаем

Наконец, из соотношения, полученного в п.2°, находим, что

Теорема доказана.

Механический смысл векторного произведения — Мегаобучалка

П. 7. Векторное произведение векторов

Определение. Тройкой векторов называется три вектора с общим началом, перечисленных в определенном порядке ( — первый, — второй, — третий) и не лежащих в одной плоскости (некомпланарных).

Определение.Тройка векторов называется «правой», если кратчайший поворот от вектора к вектору , когда смотрим с конца вектора , происходит против часовой стрелки. Если же этот поворот кажется происходящим по часовой стрелке, то тройка векторов называется «левой».

      
 
  
  
 
 

Происхождение названия: если векторы совпадают соответственно с большим, указательным и средним пальцами правой руки – тройка правая, если левой руки – тройка левая.

Смысл декартовой тройки всегда должен соответствовать правилу винта: правый винт (раскручиваем вправо, вкручиваем влево)) – тройка правая, левый винт – тройка левая.

 

Определение. Векторным произведением векторов и называется вектор , удовлетворяющий условиям:

1) , 2) , 3) образуют правую тройку. (1)

Обозначение или . Это вектор.

 

Геометрический смысл векторного произведения

Модуль векторного произведения равен площади параллелограмма, построенного на векторах и . . (2)

 
 

Механический смысл векторного произведения

1)

Пусть сила приложена к точке В. Тогда моментом силы относительно точки А называется вектор такой, что , где вектор — плечо АВ, .

 

2) Пусть материальная точка движется по окружности с центром в точке О,

— линейная скорость движения точки, — радиус-вектор точки М. Тогда угловой скоростью материальной точки называется вектор такой, что .
 
 

Свойства векторного произведения.

1. – коллинеарные векторы. (3)

Доказательство.

Доказательство необходимости: 1) Пусть – ненулевые векторы. Тогда длина векторного произведения тогда и только тогда, когда , т.е. когда . 2) Пусть среди векторов может быть нулевой вектор (или оба нулевые). По определению -вектор можно считать параллельным любому вектору, т.е. пусть .



Доказательство достаточности: 1) Пусть , причем – ненулевые векторы. Тогда длина векторного произведения , так как . 2) Пусть , причем среди векторов может быть нулевой вектор (или оба нулевые). Тогда длина векторного произведения равна нулю, так как длина — вектора равна 0. (что и треб. доказать).

Частный случай:

2. (Пояснение: из-за смены троек)

3. Скалярный квадрат векторного произведения равен квадрату модуля векторного произведения: (следует из 2-го свойства скалярного произведения)

4. Если – действительное число, то

(Пояснение: если одну из сторон параллелограмма увеличить в λ раз, не меняя ее направление, то и площадь увеличиться в λ раз).

5. ,

Перемножаем, строго соблюдая порядок.

6.

7.

 

Таблица векторного умножения ортов

Углы , , , , ,

; тогда ; длины ортов равны .

Следовательно, исходя из определения векторного произведения, можем записать, что

, , ,

.

 

Виды произведений векторов. Векторное произведение векторов, определение, свойства

На данном уроке мы рассмотрим ещё две операции с векторами: векторное произведение векторов и смешанное произведение векторов (сразу ссылка, кому нужно именно оно) . Ничего страшного, так иногда бывает, что для полного счастья, помимо скалярного произведения векторов , требуется ещё и ещё. Такая вот векторная наркомания. Может сложиться впечатление, что мы залезаем в дебри аналитической геометрии. Это не так. В данном разделе высшей математики вообще мало дров, разве что на Буратино хватит. На самом деле материал очень распространенный и простой – вряд ли сложнее, чем то же скалярное произведение , даже типовых задач поменьше будет. Главное в аналитической геометрии, как многие убедятся или уже убедились, НЕ ОШИБАТЬСЯ В ВЫЧИСЛЕНИЯХ. Повторяйте как заклинание, и будет вам счастье =)

Если векторы сверкают где-то далеко, как молнии на горизонте, не беда, начните с урока Векторы для чайников , чтобы восстановить или вновь приобрести базовые знания о векторах. Более подготовленные читатели могут знакомиться с информацией выборочно, я постарался собрать максимально полную коллекцию примеров, которые часто встречаются в практических работах

Чем вас сразу порадовать? Когда я был маленьким, то умел жонглировать двумя и даже тремя шариками. Ловко получалось. Сейчас жонглировать не придётся вообще, поскольку мы будем рассматривать только пространственные векторы , а плоские векторы с двумя координатами останутся за бортом. Почему? Такими уж родились данные действия – векторное и смешанное произведение векторов определены и работают в трёхмерном пространстве. Уже проще!

В данной операции, точно так же, как и в скалярном произведении, участвуют два вектора . Пусть это будут нетленные буквы .

Само действие обозначается следующим образом: . Существуют и другие варианты, но я привык обозначать векторное произведение векторов именно так, в квадратных скобках с крестиком.

И сразу вопрос : если в скалярном произведении векторов участвуют два вектора, и здесь тоже умножаются два вектора, тогда в чём разница ? Явная разница, прежде всего, в РЕЗУЛЬТАТЕ:

Результатом скалярного произведения векторов является ЧИСЛО:

Результатом векторного произведения векторов является ВЕКТОР : , то есть умножаем векторы и получаем снова вектор. Закрытый клуб. Собственно, отсюда и название операции. В различной учебной литературе обозначения тоже могут варьироваться, я буду использовать букву .

Определение векторного произведения

Сначала будет определение с картинкой, затем комментарии.

Определение : Векторным произведением неколлинеарных векторов , взятых в данном порядке , называется ВЕКТОР , длина которого численно равна площади параллелограмма , построенного на данных векторах; вектор ортогонален векторам , и направлен так, что базис имеет правую ориентацию:

Разбираем определение по косточкам, тут много интересного!

Итак, можно выделить следующие существенные моменты:

1) Исходные векторы , обозначенные красными стрелками, по определению не коллинеарны . Случай коллинеарных векторов будет уместно рассмотреть чуть позже.

2) Векторы взяты в строго определённом порядке : – «а» умножается на «бэ» , а не «бэ» на «а». Результатом умножения векторов является ВЕКТОР , который обозначен синим цветом. Если векторы умножить в обратном порядке, то получим равный по длине и противоположный по направлению вектор (малиновый цвет). То есть, справедливо равенство .

3) Теперь познакомимся с геометрическим смыслом векторного произведения. Это очень важный пункт! ДЛИНА синего вектора (а, значит, и малинового вектора ) численно равна ПЛОЩАДИ параллелограмма, построенного на векторах . На рисунке данный параллелограмм заштрихован чёрным цветом.

Примечание : чертёж является схематическим, и, естественно, номинальная длина векторного произведения не равна площади параллелограмма.

Вспоминаем одну из геометрических формул: площадь параллелограмма равна произведению смежных сторон на синус угла между ними . Поэтому, исходя из вышесказанного, справедлива формула вычисления ДЛИНЫ векторного произведения:

Подчёркиваю, что в формуле речь идёт о ДЛИНЕ вектора, а не о самом векторе . Каков практический смысл? А смысл таков, что в задачах аналитической геометрии площадь параллелограмма часто находят через понятие векторного произведения:

Получим вторую важную формулу. Диагональ параллелограмма (красный пунктир) делит его на два равных треугольника. Следовательно, площадь треугольника, построенного на векторах (красная штриховка), можно найти по формуле:

4) Не менее важный факт состоит в том, что вектор ортогонален векторам , то есть . Разумеется, противоположно направленный вектор (малиновая стрелка) тоже ортогонален исходным векторам .

5) Вектор направлен так, что базис имеет правую ориентацию. На уроке о переходе к новому базису я достаточно подробно рассказал об ориентации плоскости , и сейчас мы разберёмся, что такое ориентация пространства. Объяснять буду на пальцах вашей правой руки . Мысленно совместите указательный палец с вектором и средний палец с вектором . Безымянный палец и мизинец прижмите к ладони. В результате большой палец – векторное произведение будет смотреть вверх. Это и есть правоориентированный базис (на рисунке именно он). Теперь поменяйте векторы (указательный и средний пальцы ) местами, в результате большой палец развернётся, и векторное произведение уже будет смотреть вниз. Это тоже правоориентированный базис. Возможно, у вас возник вопрос: а какой базис имеет левую ориентацию? «Присвойте» тем же пальцам левой руки векторы , и полУчите левый базис и левую ориентацию пространства (в этом случае большой палец расположится по направлению нижнего вектора) . Образно говоря, данные базисы «закручивают» или ориентируют пространство в разные стороны. И это понятие не следует считать чем-то надуманным или абстрактным – так, например, ориентацию пространства меняет самое обычное зеркало, и если «вытащить отражённый объект из зазеркалья», то его в общем случае не удастся совместить с «оригиналом». Кстати, поднесите к зеркалу три пальца и проанализируйте отражение;-)

…как всё-таки хорошо, что вы теперь знаете о право- и левоориентированных базисах, ибо страшнЫ высказывания некоторых лекторов о смене ориентации =)

Векторное произведение коллинеарных векторов

Определение подробно разобрано, осталось выяснить, что происходит, когда векторы коллинеарны. Если векторы коллинеарны, то их можно расположить на одной прямой и наш параллелограмм тоже «складывается» в одну прямую. Площадь такого, как говорят математики, вырожденного параллелограмма равна нулю. Это же следует и из формулы – синус нуля или 180-ти градусов равен нулю, а значит, и площадь нулевая

Таким образом, если , то и . Обратите внимание, что само векторное произведение равно нулевому вектору, но на практике этим часто пренебрегают и пишут, что оно тоже равно нулю.

Частный случай – векторное произведение вектора на самого себя:

С помощью векторного произведения можно проверять коллинеарность трёхмерных векторов, и данную задачу среди прочих мы тоже разберём.

Для решения практических примеров может потребоваться тригонометрическая таблица , чтобы находить по ней значения синусов.

Ну что же, разжигаем огонь:

Пример 1

а) Найти длину векторного произведения векторов , если

б) Найти площадь параллелограмма, построенного на векторах , если

Решение : Нет, это не опечатка, исходные данные в пунктах условия я намеренно сделал одинаковыми. Потому что оформление решений будет отличаться!

а) По условию требуется найти длину вектора (векторного произведения). По соответствующей формуле:

Ответ :

Коль скоро спрашивалось о длине, то в ответе указываем размерность – единицы.

б) По условию требуется найти площадь параллелограмма, построенного на векторах . Площадь данного параллелограмма численно равна длине векторного произведения:

Ответ :

Обратите внимание, что в ответе о векторном произведении речи не идёт вообще, нас спрашивали о площади фигуры , соответственно, размерность – квадратные единицы.

Всегда смотрим, ЧТО требуется найти по условию, и, исходя из этого, формулируем чёткий ответ. Может показаться буквоедством, но буквоедов среди преподавателей хватает, и задание с хорошими шансами вернётся на доработку. Хотя это не особо натянутая придирка – если ответ некорректен, то складывается впечатление, что человек не разбирается в простых вещах и/или не вник в суть задания. Этот момент всегда нужно держать на контроле, решая любую задачу по высшей математике, да и по другим предметам тоже.

Куда подевалась большая буковка «эн»? В принципе, её можно было дополнительно прилепить в решение, но в целях сократить запись, я этого не сделал. Надеюсь, всем понятно, что и – это обозначение одного и того же.

Популярный пример для самостоятельного решения:

Пример 2

Найти площадь треугольника, построенного на векторах , если

Формула нахождения площади треугольника через векторное произведение дана в комментариях к определению. Решение и ответ в конце урока.

На практике задача действительно очень распространена, треугольниками вообще могут замучить.

Для решения других задач нам понадобятся:

Свойства векторного произведения векторов

Некоторые свойства векторного произведения мы уже рассмотрели, тем не менее, я их включу в данный список.

Для произвольных векторов и произвольного числа справедливы следующие свойства:

1) В других источниках информации данный пункт обычно не выделяют в свойствах, но он очень важен в практическом плане. Поэтому пусть будет.

2) – свойство тоже разобрано выше, иногда его называют антикоммутативностью . Иными словами, порядок векторов имеет значение.

3) – сочетательные или ассоциативные законы векторного произведения. Константы безпроблемно выносятся за пределы векторного произведения. Действительно, чего им там делать?

4) – распределительные или дистрибутивные законы векторного произведения. С раскрытием скобок тоже нет проблем.

В качестве демонстрации рассмотрим коротенький пример:

Пример 3

Найти , если

Решение: По условию снова требуется найти длину векторного произведения. Распишем нашу миниатюру:

(1) Согласно ассоциативным законам, выносим константы за переделы векторного произведения.

(2) Выносим константу за пределы модуля, при этом модуль «съедает» знак «минус». Длина же не может быть отрицательной.

(3) Дальнейшее понятно.

Ответ :

Пора подбросить дров в огонь:

Пример 4

Вычислить площадь треугольника, построенного на векторах , если

Решение : Площадь треугольника найдём по формуле . Загвоздка состоит в том, что векторы «цэ» и «дэ» сами представлены в виде сумм векторов. Алгоритм здесь стандартен и чем-то напоминает примеры № 3 и 4 урока Скалярное произведение векторов . Решение для ясности разобьём на три этапа:

1) На первом шаге выразим векторное произведение через векторное произведение , по сути, выразим вектор через вектор . О длинах пока ни слова!

(1) Подставляем выражения векторов .

(2) Используя дистрибутивные законы, раскрываем скобки по правилу умножения многочленов.

(3) Используя ассоциативные законы, выносим все константы за пределы векторных произведений. При маломальском опыте действия 2 и 3 можно выполнять одновременно.

(4) Первое и последнее слагаемое равно нулю (нулевому вектору) благодаря приятному свойству . Во втором слагаемом используем свойство антикоммутативности векторного произведения:

(5) Приводим подобные слагаемые.

В результате вектор оказался выражен через вектор, чего и требовалось достичь:

2) На втором шаге найдем длину нужного нам векторного произведения. Данное действие напоминает Пример 3:

3) Найдём площадь искомого треугольника:

Этапы 2-3 решения можно было оформить и одной строкой.

Ответ :

Рассмотренная задача достаточно распространена в контрольных работах, вот пример для самостоятельного решения:

Пример 5

Найти , если

Краткое решение и ответ в конце урока. Посмотрим, насколько вы были внимательны при изучении предыдущих примеров;-)

Векторное произведение векторов в координатах , заданных в ортонормированном базисе , выражается формулой :

Формула и правда простецкая: в верхнюю строку определителя записываем координатные векторы, во вторую и третью строки «укладываем» координаты векторов , причём укладываем в строгом порядке – сначала координаты вектора «вэ», затем координаты вектора «дубль-вэ». Если векторы нужно умножить в другом порядке, то и строки следует поменять местами:

Пример 10

Проверить, будут ли коллинеарны следующие векторы пространства:
а)
б)

Решение : Проверка основана на одном из утверждений данного урока: если векторы коллинеарны, то их векторное произведение равно нулю (нулевому вектору): .

а) Найдём векторное произведение:

Таким образом, векторы не коллинеарны.

б) Найдём векторное произведение:

Ответ : а) не коллинеарны, б)

Вот, пожалуй, и все основные сведения о векторном произведении векторов.

Данный раздел будет не очень большим, так как задач, где используется смешанное произведение векторов, немного. Фактически всё будет упираться в определение, геометрический смысл и пару рабочих формул.

Смешанное произведение векторов – это произведение трёх векторов :

Вот так вот они выстроились паровозиком и ждут, не дождутся, когда их вычислят.

Сначала опять определение и картинка:

Определение : Смешанным произведением некомпланарных векторов , взятых в данном порядке , называется объём параллелепипеда , построенного на данных векторах, снабжённый знаком «+», если базис правый, и знаком «–», если базис левый.

Выполним рисунок. Невидимые нам линии прочерчены пунктиром:

Погружаемся в определение:

2) Векторы взяты в определённом порядке , то есть перестановка векторов в произведении , как вы догадываетесь, не проходит без последствий.

3) Перед тем, как прокомментировать геометрический смысл, отмечу очевидный факт: смешанное произведение векторов является ЧИСЛОМ : . В учебной литературе оформление может быть несколько другим, я привык обозначать смешанное произведение через , а результат вычислений буквой «пэ».

По определению смешанное произведение – это объем параллелепипеда , построенного на векторах (фигура прочерчена красными векторами и линиями чёрного цвета). То есть, число равно объему данного параллелепипеда.

Примечание : чертёж является схематическим.

4) Не будем заново париться с понятием ориентации базиса и пространства. Смысл заключительной части состоит в том, что к объёму может добавляться знак минус. Простыми словами, смешанное произведение может быть отрицательным: .

Непосредственно из определения следует формула вычисления объема параллелепипеда, построенного на векторах .

Векторное произведение — это псевдовектор, перпендикулярный плоскости, построенной по двум сомножителям, являющийся результатом бинарной операции «векторное умножение» над векторами в трёхмерном Евклидовом пространстве. Векторное произведение не обладает свойствами коммутативности и ассоциативности (является антикоммутативным) и, в отличие от скалярного произведения векторов, является вектором. Широко используется во многих технических и физических приложениях. Например, момент импульса и сила Лоренца математически записываются в виде векторного произведения. Векторное произведение полезно для «измерения» перпендикулярности векторов — модуль векторного произведения двух векторов равен произведению их модулей, если они перпендикулярны, и уменьшается до нуля, если векторы параллельны либо антипараллельны.

Определить векторное произведение можно по-разному, и теоретически, в пространстве любой размерности n можно вычислить произведение n-1 векторов, получив при этом единственный вектор, перпендикулярный к ним всем. Но если произведение ограничить нетривиальными бинарными произведениями с векторным результатами, то традиционное векторное произведение определено только в трёхмерном и семимерном пространствах. Результат векторного произведения, как и скалярного, зависит от метрики Евклидова пространства.

В отличие от формулы для вычисления по координатам векторов скалярного произведения в трёхмерной прямоугольной системе координат, формула для векторного произведения зависит от ориентации прямоугольной системы координат или, иначе, её «хиральности».

Определение:
Векторным произведением вектора a на вектор b в пространстве R 3 называется вектор c , удовлетворяющий следующим требованиям:
длина вектора c равна произведению длин векторов a и b на синус угла φ между ними:
|c|=|a||b|sin φ;
вектор c ортогонален каждому из векторов a и b;
вектор c направлен так, что тройка векторов abc является правой;
в случае пространства R7 требуется ассоциативность тройки векторов a,b,c.
Обозначение:
c===a × b


Рис. 1. Площадь параллелограмма равна модулю векторного произведения

Геометрические свойства векторного произведения :
Необходимым и достаточным условием коллинеарности двух ненулевых векторов является равенство нулю их векторного произведения.

Модуль векторного произведения равняется площади S параллелограмма, построенного на приведённых к общему началу векторах a и b (см. рис.1).

Если e — единичный вектор, ортогональный векторам a и b и выбранный так, что тройка a,b,e — правая, а S — площадь параллелограмма, построенного на них (приведённых к общему началу), то для векторного произведения справедлива формула:
=S e


Рис.2. Объём параллелепипеда при использовании векторного и скалярного произведения векторов; пунктирные линии показывают проекции вектора c на a × b и вектора a на b × c, первым шагом является нахождение скалярных произведений

Если c — какой-нибудь вектор, π — любая плоскость, содержащая этот вектор, e — единичный вектор, лежащий в плоскости π и ортогональный к c,g — единичный вектор, ортогональный к плоскости π и направленный так, что тройка векторов ecg является правой, то для любого лежащего в плоскости π вектора a справедлива формула:
=Pr e a |c|g
где Pr e a проекция вектора e на a
|c|-модуль вектора с

При использовании векторного и скалярного произведений можно высчитать объём параллелепипеда, построенного на приведённых к общему началу векторах a, b и c . Такое произведение трех векторов называется смешанным.
V=|a (b×c)|
На рисунке показано, что этот объём может быть найден двумя способами: геометрический результат сохраняется даже при замене «скалярного» и «векторного» произведений местами:
V=a×b c=a b×c

Величина векторного произведения зависит от синуса угла между изначальными векторами, поэтому векторное произведение может восприниматься как степень «перпендикулярности» векторов также, как и скалярное произведение может рассматриваться как степень «параллельности». Векторное произведение двух единичных векторов равно 1 (единичному вектору), если изначальные векторы перпендикулярны, и равно 0 (нулевому вектору), если векторы параллельны либо антипараллельны.

Выражение для векторного произведения в декартовых координатах
Если два вектора a и b определены своими прямоугольными декартовыми координатами, а говоря точнее — представлены в ортонормированном базисе
a=(a x ,a y ,a z)
b=(b x ,b y ,b z)
а система координат правая, то их векторное произведение имеет вид
=(a y b z -a z b y ,a z b x -a x b z ,a x b y -a y b x)
Для запоминания этой формулы:
i =∑ε ijk a j b k
где ε ijk — символ Леви-Чивиты.

Определение. Векторным произведением вектора а (множимое) на не коллинеарный ему вектор (множитель) называется третий вектор с (произведение), который строится следующим образом:

1) его модуль численно равен площади параллелограмма на рис. 155), построенного на векторах т. е. он равен направление перпендикулярно плоскости упомянутого параллелограмма;

3) при этом направление вектора с выбирается (из двух возможных) так, чтобы векторы с составляли правую систему (§ 110).

Обозначение: или

Дополнение к определению. Если векторы коллинеарны, то фигуре считая ее (условно) параллелограммом, естественно приписать нулевую площадь. Поэтому векторное произведение коллинеарных векторов считается равным нуль-вектору.

Поскольку нуль-вектору можно приписать любое направление, это соглашение не противоречит пунктам 2 и 3 определения.

Замечание 1. В термине «векторное произведение» первое слово указывает на то, что результат действия есть вектор (в противоположность скалярному произведению; ср. § 104, замечание 1).

Пример 1. Найти векторное произведение где основные векторы правой системы координат (рис. 156).

1. Так как длины основных векторов равны единице масштаба, то площадь параллелограмма (квадрата) численно равна единице. Значит, модуль векторного произведения равен единице.

2. Так как перпендикуляр к плоскости есть ось то искомое векторное произведение есть вектор, коллинеарный вектору к; а так как оба они имеют модуль 1, то искомое векторное произведение равно либо k, либо -k.

3. Из этих двух возможных векторов надо выбрать первый, так как векторы к образуют правую систему (а векторы левую).

Пример 2. Найти векторное произведение

Решение. Как в примере 1, заключаем, что вектор равен либо k, либо -k. Но теперь надо выбрать -k, так как векторы образуют правую систему (а векторы левую). Итак,

Пример 3. Векторы имеют длины, соответственно равные 80 и 50 см, и образуют угол 30°. Приняв за единицу длины метр, найти длину векторного произведения а

Решение. Площадь параллелограмма, построенного на векторах равна Длина искомого векторного произведения равна

Пример 4. Найти длину векторного произведения тех же векторов, приняв за единицу длины сантиметр.

Решение. Так как площадь параллелограмма, построенного на векторах равна то длина векторного произведения равна 2000 см, т. е.

Из сравнения примеров 3 и 4 видно, что длина вектора зависит не только от длин сомножителей но также и от выбора единицы длины.

Физический смысл векторного произведения. Из многочисленных физических величин, изображаемых векторным произведением, рассмотрим только момент силы.

Пусть А есть точка приложения силы Моментом силы относителько точки О называется векторное произведение Так как модуль этого векторного произведения численно равен площади параллелограмма (рис. 157), то модуль момента равняется произведению основания на высоту т. е. силе, умноженной на расстояние от точки О до прямой, вдоль которой действует сила.

В механике доказывается, что для равновесия твердого тела необходимо, чтобы равнялась нулю не только сумма векторов , представляющих силы, приложенные к телу, но также и сумма моментов сил. j ;

2) |k |=1, но | i x j | = |i | |J | sin(90°)=1;

3) векторы i , j и k образуют правую тройку (см. рис. 16).

7.2. Свойства векторного произведения

1. При перестановке сомножителей векторное произведение меняет знак, т.е. а хb =(b хa ) (см. рис. 19).

Векторы а хb и b ха коллинеарны, имеют одинаковые модули (площадь параллелограмма остается неизменной), но противоположно направлены (тройки а , b , а хb и a , b , b x a противоположной ориентации). Стало быть a xb = -(b xa ).

2. Векторное произведение обладает сочетательным свойством относительно скалярного множителя, т. е. l (а хb ) = (l а ) х b = а х (l b ).

Пусть l >0. Вектор l (а хb ) перпендикулярен векторам а и b . Вектор ( l а )хb также перпендикулярен векторам а и b (векторы а , l а лежат в одной плоскости). Значит, векторы l (а хb ) и ( l а )хb коллинеарны. Очевидно, что и направления их совпадают. Имеют одинаковую длину:

Поэтому l (a хb )= l а хb . Аналогично доказывается при l

3. Два ненулевых вектора а и b коллинеарны тогда и только тогда, когда их векторное произведение равно нулевому вектору, т. е. а ||b а хb =0 .

В частности, i *i =j *j =k *k =0 .

4. Векторное произведение обладает распределительным свойством:

(a +b ) хс = а хс +b хс .

Примем без доказательства.

7.3. Выражение векторного произведения через координаты

Мы будем использовать таблицу векторного произведения векторов i , j и k :

если направление кратчайшего пути от первого вектора к второму совпадает с направлением стрелки, то произведение равно третьему вектору, если не совпадает — третий вектор берется со знаком «минус».

Пусть заданы два вектора а =а х i +a y j +a z k и b =b x i +b y j +b z k . Найдем векторное произведение этих векторов, перемножая их как многочлены (согласно свойств векторного произведения):



Полученную формулу можно записать еще короче:

так как правая часть равенства (7.1) соответствует разложению определителя третьего порядка по элементам первой строки.Равенство (7.2) легко запоминается.

7.4. Некоторые приложения векторного произведения

Установление коллинеарности векторов

Нахождение площади параллелограмма и треугольника

Согласно определению векторного произведения векторов а и b |а хb | = |а | * |b |sin g , т. е. S пар = |а х b |. И, значит, D S =1/2|а х b |.

Определение момента силы относительно точки

Пусть в точке А приложена сила F =АВ и пусть О — некоторая точка пространства (см. рис. 20).

Из физики известно, что моментом си лы F относительно точки О называется вектор М , который проходит через точку О и:

1) перпендикулярен плоскости, проходящей через точки О, А, В;

2) численно равен произведению силы на плечо

3) образует правую тройку с векторами ОА и A В .

Стало быть, М =ОА х F .

Нахождение линейной скорости вращения

Скорость v точки М твердого тела, вращающегося с угловой скоростью w вокруг неподвижной оси, определяется формулой Эйлера v =w хr , где r =ОМ , где О-некоторая неподвижная точка оси (см. рис. 21).

Угол между векторами

Для того чтобы мы могли ввести понятие векторного произведения двух векторов, нужно сначала разобраться с таким понятие, как угол между этими векторами.

Пусть нам даны два вектора $\overline{α}$ и $\overline{β}$. Возьмем в пространстве какую-либо точку $O$ и отложим от нее векторы $\overline{α}=\overline{OA}$ и $\overline{β}=\overline{OB}$, тогда угол $AOB$ будет называться углом между этими векторами (рис. 1).

Обозначение: $∠(\overline{α},\overline{β})$

Понятие векторного произведения векторов и формула нахождения

Определение 1

Векторным произведением двух векторов называется вектор, перпендикулярный обоим данным векторам, и его длина будет равняться произведению длин этих векторов с синусом угла между данными векторами, а также этот вектор с двумя начальными имеют туже ориентацию, как и декартова система координат.\circ=4\cdot 3\cdot 1=12$

Ответ: $12$.

Вычисление векторного произведения по координатам векторов

Из определения 1 сразу же вытекает и способ нахождения векторного произведения для двух векторов. Поскольку вектор кроме значения имеет еще и направление, находить его только при помощи скалярной величины невозможно. Но помимо него существует еще способ нахождения с помощью координат данных нам векторов.

Пусть нам даны векторы $\overline{α}$ и $\overline{β}$, которые будут иметь координаты $(α_1,α_2,α_3)$ и $(β_1,β_2,β_3)$, соответственно. Тогда вектор векторного произведения (а именно его координаты) можно найти по следующей формуле:

$\overline{α}х\overline{β}=\begin{vmatrix}\overline{i}&\overline{j}&\overline{k}\\α_1&α_2&α_3\\β_1&β_2&β_3\end{vmatrix}$

Иначе, раскрывая определитель, получим следующие координаты

$\overline{α}х\overline{β}=(α_2 β_3-α_3 β_2,α_3 β_1-α_1 β_3,α_1 β_2-α_2 β_1)$

Пример 2

Найти вектор векторного произведения коллинеарных векторов $\overline{α}$ и $\overline{β}$ с координатами $(0,3,3)$ и $(-1,2,6)$.

Решение .

Воспользуемся формулой, приведенной выше. Получим

$\overline{α}х\overline{β}=\begin{vmatrix}\overline{i}&\overline{j}&\overline{k}\\0&3&3\\-1&2&6\end{vmatrix}=(18-6)\overline{i}-(0+3)\overline{j}+(0+3)\overline{k}=12\overline{i}-3\overline{j}+3\overline{k}=(12,-3,3)$

Ответ: $(12,-3,3)$.

Свойства векторного произведения векторов

Для произвольных смешанных трех векторов $\overline{α}$, $\overline{β}$ и $\overline{γ}$, а также $r∈R$ справедливы следующие свойства:

Пример 3

Найдите площадь параллелограмма, вершины которого имеют координаты $(3,0,0)$, $(0,0,0)$, $(0,8,0)$ и $(3,8,0)$.

Решение .

Вначале изобразим данный параллелограмм в координатном пространстве (рис.5):

Рисунок 5. Параллелограмм в координатном пространстве. Автор24 — интернет-биржа студенческих работ

Видим, что две стороны этого параллелограмма построены с помощью коллинеарных векторов с координатами $\overline{α}=(3,0,0)$ и $\overline{β}=(0,8,0)$.2}=24$

О возможности определения векторного произведения двух векторов в многомерном пространстве Текст научной статьи по специальности «Математика»

УДК 514.742.24

О ВОЗМОЖНОСТИ ОПРЕДЕЛЕНИЯ ВЕКТОРНОГО ПРОИЗВЕДЕНИЯ ДВУХ ВЕКТОРОВ В МНОГОМЕРНОМ

ПРОСТРАНСТВЕ

ON POSSIBILITY OF VECTOR PRODUCT DETERMINATION OF TWO VECTORS IN MULTIDIMENSIONAL SPACE

И.П. Попов I.P. Popov

Курганский государственный университет, Россия, 640020, г. Курган, ул.

Советская, 63/4

Kurgan state university, 63/4 Sovetskaja St, Kurgan, 640020, Russia

E-mail: [email protected]

Аннотация

Целью работы является определение векторного произведения двух векторов c = [a, b] в n-мерном евклидовом пространстве при n > 3, которое удовлетворяет общепринятому инвариантному определению, в соответствии с которым оно является вектором, модуль которого равен площади параллелограмма, построенного на векторах a и b, его направление перпендикулярно обоим векторам и векторы a, b и c образуют правую тройку векторов. В работе применяются ортонормированные базисы. Доказывается, что для двух линейно независимых векторов a и b в Rn существует их векторное произведение. Вводится понятие m-расщепления и симметричного m-расщепления базисных векторов, под которыми понимается трансформация Rn в Rn+m-1 путем замены ei на m векторов ei1,…,e . «e™, ортогональных друг другу и всем другим базисным векторам исходного базиса. Решается некоторым образом обратная задача -при известном векторном произведении определение координат всех трех векторов в Rn. Устанавливается условие, в соответствии с которым векторное произведение c = [a, b] в Rn лежит на одной прямой с проекцией суммы базисных ортов на (n — 2)-плоскость, перпендикулярную векторам a и b.

Abstract

The aim of the paper is to define the vector product of two vectors c = [a, b] in n-dimensional Euclidean space for n > 3. Orthonormal bases are used in this paper. It is proved that for two linearly independent vectors a and b in Rn exists their vector product. We introduce the notion of m-splitting and symmetric m-splitting of basis vectors, by which is meant a transformation Rn in Rn+m-1 by replacing ei by m vectors ei1,…, ej e™ orthogonal to each other and to all other basis vectors of the original basis. The inverse problem is solved in some way — for a known vector product the definition of the coordinates of all three vectors in Rn. A condition is established in accordance with which the vector product c = [a, b] in Rn lies on one line with the projection of the sum of the basis vectors to the (n — 2)-plane perpendicular to the vectors a and b.

Ключевые слова: векторное произведение, многомерное пространство, базис, расщепление.

Keywords: vector product, multidimensional space, basis, splitting.

Целью работы является установление векторного произведения двух векторов с = [а, Ь] в п-мерном евклидовом пространстве при п > 3, которое, безусловно, удовлетворяет общепринятому инвариантному определению, в соответствии с которым оно является вектором, модуль которого равен площади параллелограмма, построенного на векторах а и Ь, его направление перпендикулярно обоим векторам и векторы а, Ь и с образуют правую тройку векторов.

Далее применяются ортонормированные базисы [1].

Теорема существования

Теорема 1. Для двух линейно независимых векторов a и b в Rn существует их векторное произведение c = [a, b].

Доказательство. Три линейно независимых вектора a, b и g имеют инвариантное описание, включающее в себя длины векторов, углы между ними и их взаимную ориентацию. Для каждого из этих трех векторов однозначно определена их проекция на любой другой вектор. Другими словами, определены их попарные скалярные произведения.

В этой связи векторы a, b и g могут иметь однозначное координатное описание в базисах любой размерности, начиная с 3 (пассивная точка зрения (alias)). При координатном описании они сохраняют размеры, углы между ними и взаимную ориентацию, поскольку в базисе любой размерности их попарные скалярные произведения остаются неизменными. Другими словами, координатное описание той или иной размерности при пассивной точке зрения не меняет сущность векторов и их отношений друг к другу. Следовательно, если в качестве вектора g рассматривать вектор c, являющийся при инвариантном описании векторным произведением векторов a и b, то его сущность в этом качестве не изменится при координатном описании в Rn. Теорема доказана.

Расщепление базисных векторов

Определение 1. m-расщеплением базисного вектора ei является трансформация Rn в Rn+m-1 путем замены ei на m векторов e’1,…,e . «e,m, ортогональных друг другу и всем другим базисным векторам исходного базиса, при этом

m

e = У кe

1=1

где kij — направляющие косинусы ei в базисе e’1,…,eij. «e,m.

Выбор направляющих косинусов kij может быть сопряжен с произволом. Произвол минимизируется при симметричном m-расщеплении.

Определение 2. Симметричное m-расщепление базисного вектора — это m-расщепление, при котором

um

Vj e [1, m]\ kj =J-L

m. Г 01

а = 3 Ь = 5

,0 у , 0 у

Координаты векторного произведения с = [а, Ь] равны

Г о 1 о

VI0 у

Базисный вектор ез подвергается симметричному 2-расщеплению. В образовавшемся Я4 координаты векторов равны

Г 21 Г 01 Г 0

3 1ь = 5 0

а = ‘с =

0 0 7,071

V 0 у V 0 у ч 7,071

Произвольная квадратная матрица отображения

т =

‘ 0,497 0,628 0,287 — 0,527″

0,47 0,22 — 0,814 0,262

-0,171 0,604 0,296 0,72

ч 0,709 — 0,439 0,41 0,369у

позволяет получить координаты всех трех векторов в другом базисе этой же размерности

2 гт 1

а = т • а =

Г2,8761 1,599 1,47 0,101

2Ь = Т • Ь =

Г 3,138 1 1,099 3,02 -2,197

2 гр 1

с = Т • с =

Г-1,695 1 -3,902 7,184 5,503

с =

Здесь с = [а, Ь].

Ориентация векторного произведения

В Я3 вектор с = [а, Ь] лежит на линии пересечения плоскостей, нормалями которых являются векторы а и Ь или в терминах многомерного пространства — в 1-плоскости, образованной пересечением двух 2-плоскостей. В этой 1-плоскости можно построить два противоположно направленных вектора, величина которых равна модулю векторного произведения, и ортогональных векторам а и Ь. Формально концы этих векторов образуют в 1-плоскости 0-сферу.

В Я4 векторы а и Ь служат нормалями двум 3-плоскостям, пересечением которых является 2-плоскость, все векторы которой ортогональны векторам а и Ь. Концы векторов, величина которых равна модулю векторного произведения, образуют в этой 2-плоскости 1-сферу (окружность).

В Яп векторы а и Ь служат нормалями двум (п — 1)-плоскостям, пересечением которых является (п — 2)-плоскость. Концы векторов, величина которых равна модулю векторного произведения, и ортогональных векторам а и Ь, образуют в этой (п — 2)-плоскости (п — 3)-сферу.

И в Я3 и в Яп имеет место неоднозначность при выборе направления векторного произведения двух векторов. В Я3 приходится выбирать из векторов, ограниченных 0-сферой, в Яп — из векторов, ограниченных (п — 3)-сферой.

В Я3 неоднозначность преодолевается постулированием — в качестве направления с выбирается вектор правый относительно а и Ь.

Неоднозначность в Яп может быть преодолена также, как и в Я3 — выбором одного наиболее подходящего варианта.

По аналогии с взаимным расположением вектора с и вектора, являющегося суммой базисных ортов, имеющем место для частного случая (1), для произвольного базиса можно принять следующее

Условие 1. 021 122 — 02,. — 02j. — 02и

0,i 0„ — cos ф — sin ф — 0ir

0jl 0j2—-sin ф — cosф— 0j.

V 0„1 0„2 — 0„, — 0„j — 1„„ J

(3)

При этом cosф и sinф находятся из условия

dt cos ф + dj sin ф = 1d¡ -d, sin ф + d : cos ф = 1J :

1d¡ = 0

cos ф = —

— sin ф= г—

Jdi + dj Jdi + d

если

d

d

При этом

с

Все другие координаты остаются без изменения. = 0, т.е. сумма базисных ортов б линейно зависима от векторов а и Ь, то их векторное произведение в соответствии с условием 1 неопределимо.

Пример 2. В Я4 по известным значениям а и Ь отыскать с = [а, Ь].

а =

Г 3,37 1 2,762 -2,395 -0,532

Ь =

Г 3,2891 1,539 -5,697 1,834

т =

Г0,815 0 — 0,579 01

0 1 0 0

0,579 0 0,815 0

0 0 0 1

1а = Та =

Г 4,1341 2,762 0

-0,532

Т = 1 2

Т3 =

Г 0,831 0,556 0 01

-0,556 0,831 0 0

0 0 1 0

0 0 0 1

Г0,994 0 0 — 0,1061

0 1 0 0

0 0 1 0

ч0,106 0 0 0,994у

2 гр 1

а = Т2 • а =

Г 4,9721 0 0

3 т 2

а = Т • а =

V-0,532

Г 51

0

0

V 0 У

Т = ТТТ =

Г 0,674 0,552 — 0,479 — 0,106 1

-0,453 0,831 0,322 0

0,579 0 0,815 0

0,072 0,059 — 0,051 0,994

3Ь = ТЪ1Ь =

Г 5,6 1 -2,043 -2,738 2,443

Т4 =

Г 1 о о о ^

0 — 0,641 0 0,767

0 0 1 0

0 — 0,767 0 — 0,641

4Ь = Т4 • 3Ь =

Г 5,6 ^ 3,185 -2,738 ч 0 /

Т5 =

Г 1 0 0 0 ^

0 0,758 — 0,652 0

0 0,652 0,758 0

0 0 0 1

5Ь = Т • 4Ь =

Г 5,6 ^ 4,2 0

ч 0 /

Т51 = Т5Т4Т31 = Т5Т4Т3Т2Т =

Г 0,674 0,552 — 0,479 — 0,106 ^

-0,115 — 0,37 — 0,718 0,578

0,665 — 0,318 0,458 0,497

ч 0,301 — 0,676 — 0,214 — 0,638/

5„ 5„ 5„ 5,

Сумма ортов исходного базиса e2, eз, е4 в базисе e2, eз, е4 имеет координаты

5 = Т515 = Т51

т 1 1 1

Г 0,641 ^ -0,625 1,302

ч-1,226 /

Те =

Г1 0 0 0 ^

0 1 0 0

0 0 0,0297 — 0,9996

0 0 0,9996 0,0297

6 5 = Т6 • 5 5 =

Г 0,641 ^ -0,625 1,265

ч 1,265,

Матрица перехода от исходного базиса e2, eз, е4 к базису

6е 6е 6е 6е

е1, е2, е3, е4

равна

Т61 = ТТ = Т6Т5Т4Т3Т2Т =

Г 0,674 0,552 — 0,479 — 0,106 ^

-0,115 — 0,37 — 0,718 0,578

-0,281 0,666 0,228 0,652

0,673 — 0,338 0,451 0,478

Очевидно, что

6 ГГ1 3 6» ГТ1 1 5» 6 ГТ1

а = Т61а = а Ь = Т61Ь = Ь 5 = Т615

6„ 6„ б 6„

Координаты вектора с = [а, Ь] в последнем базисе e2, eз, е4 в соответствии с (4) равны

Г 0 ^ 0

14,849 ч14,849,

6

с =

Координаты вектора с = [а, Ь] в исходном базисе e2, eз, е4 равны

‘ 5,822 ^

, 6 4,869 с = г-1 • с =

61 10,081

v16,786,

Замечание 2. Порядок обнуления координат и, следовательно, значения промежуточных матриц могут быть иными. При этом нетрудно убедиться, что итоговая матрица T (T61) и значение вектора c в исходном базисе не изменяются.

Список литературы References

1. Попов И.П. 2017. Операторы типа набла: поверхностный, нулевой и мнимый нулевой. Научные ведомости Белгородского государственного университета. Математика. Физика. № 6(255), выпуск 46: 44-53.

Popov I.P. 2017. Operators type of nabla: surface, zero and zero imaginary. Nauchnye vedomosti Belgorodskogo gosudarstvennogo universiteta. Matematika. Fizika [Scientific statements Belgorod State University. Mathematics. Physics]. № 6(255), 46: 44-53.

2. Чурсанова А.С. 2017. Оценка собственных значений матриц. Научные ведомости Белгородского государственного университета. Математика. Физика. № 6(255), выпуск 46: 59-61.

Chursanova A.S. 2017. The estimate of matrix eigenvalues. Nauchnye vedomosti Belgorodskogo gosudarstvennogo universiteta. Matematika. Fizika [Scientific statements Belgorod State University. Mathematics. Physics]. № 6(255), 46: 5961.

Векторное произведение в цилиндрических координатах. Векторное произведение

Векторное произведение — это псевдовектор, перпендикулярный плоскости, построенной по двум сомножителям, являющийся результатом бинарной операции «векторное умножение» над векторами в трёхмерном Евклидовом пространстве. Векторное произведение не обладает свойствами коммутативности и ассоциативности (является антикоммутативным) и, в отличие от скалярного произведения векторов, является вектором. Широко используется во многих технических и физических приложениях. Например, момент импульса и сила Лоренца математически записываются в виде векторного произведения. Векторное произведение полезно для «измерения» перпендикулярности векторов — модуль векторного произведения двух векторов равен произведению их модулей, если они перпендикулярны, и уменьшается до нуля, если векторы параллельны либо антипараллельны.

Определить векторное произведение можно по-разному, и теоретически, в пространстве любой размерности n можно вычислить произведение n-1 векторов, получив при этом единственный вектор, перпендикулярный к ним всем. Но если произведение ограничить нетривиальными бинарными произведениями с векторным результатами, то традиционное векторное произведение определено только в трёхмерном и семимерном пространствах. Результат векторного произведения, как и скалярного, зависит от метрики Евклидова пространства.

В отличие от формулы для вычисления по координатам векторов скалярного произведения в трёхмерной прямоугольной системе координат, формула для векторного произведения зависит от ориентации прямоугольной системы координат или, иначе, её «хиральности».

Определение:
Векторным произведением вектора a на вектор b в пространстве R 3 называется вектор c , удовлетворяющий следующим требованиям:
длина вектора c равна произведению длин векторов a и b на синус угла φ между ними:
|c|=|a||b|sin φ;
вектор c ортогонален каждому из векторов a и b;
вектор c направлен так, что тройка векторов abc является правой;
в случае пространства R7 требуется ассоциативность тройки векторов a,b,c.
Обозначение:
c===a × b


Рис. 1. Площадь параллелограмма равна модулю векторного произведения

Геометрические свойства векторного произведения :
Необходимым и достаточным условием коллинеарности двух ненулевых векторов является равенство нулю их векторного произведения.

Модуль векторного произведения равняется площади S параллелограмма, построенного на приведённых к общему началу векторах a и b (см. рис.1).

Если e — единичный вектор, ортогональный векторам a и b и выбранный так, что тройка a,b,e — правая, а S — площадь параллелограмма, построенного на них (приведённых к общему началу), то для векторного произведения справедлива формула:
=S e


Рис.2. Объём параллелепипеда при использовании векторного и скалярного произведения векторов; пунктирные линии показывают проекции вектора c на a × b и вектора a на b × c, первым шагом является нахождение скалярных произведений

Если c — какой-нибудь вектор, π — любая плоскость, содержащая этот вектор, e — единичный вектор, лежащий в плоскости π и ортогональный к c,g — единичный вектор, ортогональный к плоскости π и направленный так, что тройка векторов ecg является правой, то для любого лежащего в плоскости π вектора a справедлива формула:
=Pr e a |c|g
где Pr e a проекция вектора e на a
|c|-модуль вектора с

При использовании векторного и скалярного произведений можно высчитать объём параллелепипеда, построенного на приведённых к общему началу векторах a, b и c . Такое произведение трех векторов называется смешанным.
V=|a (b×c)|
На рисунке показано, что этот объём может быть найден двумя способами: геометрический результат сохраняется даже при замене «скалярного» и «векторного» произведений местами:
V=a×b c=a b×c

Величина векторного произведения зависит от синуса угла между изначальными векторами, поэтому векторное произведение может восприниматься как степень «перпендикулярности» векторов также, как и скалярное произведение может рассматриваться как степень «параллельности». Векторное произведение двух единичных векторов равно 1 (единичному вектору), если изначальные векторы перпендикулярны, и равно 0 (нулевому вектору), если векторы параллельны либо антипараллельны.

Выражение для векторного произведения в декартовых координатах
Если два вектора a и b определены своими прямоугольными декартовыми координатами, а говоря точнее — представлены в ортонормированном базисе
a=(a x ,a y ,a z)
b=(b x ,b y ,b z)
а система координат правая, то их векторное произведение имеет вид
=(a y b z -a z b y ,a z b x -a x b z ,a x b y -a y b x)
Для запоминания этой формулы:
i =∑ε ijk a j b k
где ε ijk — символ Леви-Чивиты.

7.1. Определение векторного произведения

Три некомпланарных вектора a , b и с , взятые в указанном порядке, образуют правую тройку, если с конца третьего вектора с кратчайший поворот от первого вектора а ко второму вектору b виден совершающимся против часовой стрелки, и левую, если по часовой (см. j ;

2) |k |=1, но | i x j | = |i | |J | sin(90°)=1;

3) векторы i , j и k образуют правую тройку (см. рис. 16).

7.2. Свойства векторного произведения

1. При перестановке сомножителей векторное произведение меняет знак, т.е. а хb =(b хa ) (см. рис. 19).

Векторы а хb и b ха коллинеарны, имеют одинаковые модули (площадь параллелограмма остается неизменной), но противоположно направлены (тройки а , b , а хb и a , b , b x a противоположной ориентации). Стало быть a xb = -(b xa ).

2. Векторное произведение обладает сочетательным свойством относительно скалярного множителя, т. е. l (а хb ) = (l а ) х b = а х (l b ).

Пусть l >0. Вектор l (а хb ) перпендикулярен векторам а и b . Вектор ( l а )хb также перпендикулярен векторам а и b (векторы а , l а лежат в одной плоскости). Значит, векторы l (а хb ) и ( l а )хb коллинеарны. Очевидно, что и направления их совпадают. Имеют одинаковую длину:

Поэтому l (a хb )= l а хb . Аналогично доказывается при l

3. Два ненулевых вектора а и b коллинеарны тогда и только тогда, когда их векторное произведение равно нулевому вектору, т. е. а ||b а хb =0 .

В частности, i *i =j *j =k *k =0 .

4. Векторное произведение обладает распределительным свойством:

(a +b ) хс = а хс +b хс .

Примем без доказательства.

7.3. Выражение векторного произведения через координаты

Мы будем использовать таблицу векторного произведения векторов i , j и k :

если направление кратчайшего пути от первого вектора к второму совпадает с направлением стрелки, то произведение равно третьему вектору, если не совпадает — третий вектор берется со знаком «минус».

Пусть заданы два вектора а =а х i +a y j +a z k и b =b x i +b y j +b z k . Найдем векторное произведение этих векторов, перемножая их как многочлены (согласно свойств векторного произведения):



Полученную формулу можно записать еще короче:

так как правая часть равенства (7.1) соответствует разложению определителя третьего порядка по элементам первой строки.Равенство (7.2) легко запоминается.

7.4. Некоторые приложения векторного произведения

Установление коллинеарности векторов

Нахождение площади параллелограмма и треугольника

Согласно определению векторного произведения векторов а и b |а хb | = |а | * |b |sin g , т. е. S пар = |а х b |. И, значит, D S =1/2|а х b |.

Определение момента силы относительно точки

Пусть в точке А приложена сила F =АВ и пусть О — некоторая точка пространства (см. рис. 20).

Из физики известно, что моментом си лы F относительно точки О называется вектор М , который проходит через точку О и:

1) перпендикулярен плоскости, проходящей через точки О, А, В;

2) численно равен произведению силы на плечо

3) образует правую тройку с векторами ОА и A В .

Стало быть, М =ОА х F .

Нахождение линейной скорости вращения

Скорость v точки М твердого тела, вращающегося с угловой скоростью w вокруг неподвижной оси, определяется формулой Эйлера v =w хr , где r =ОМ , где О-некоторая неподвижная точка оси (см. рис. 21).

Угол между векторами

Для того чтобы мы могли ввести понятие векторного произведения двух векторов, нужно сначала разобраться с таким понятие, как угол между этими векторами.

Пусть нам даны два вектора $\overline{α}$ и $\overline{β}$. Возьмем в пространстве какую-либо точку $O$ и отложим от нее векторы $\overline{α}=\overline{OA}$ и $\overline{β}=\overline{OB}$, тогда угол $AOB$ будет называться углом между этими векторами (рис. 1).

Обозначение: $∠(\overline{α},\overline{β})$

Понятие векторного произведения векторов и формула нахождения

Определение 1

Векторным произведением двух векторов называется вектор, перпендикулярный обоим данным векторам, и его длина будет равняться произведению длин этих векторов с синусом угла между данными векторами, а также этот вектор с двумя начальными имеют туже ориентацию, как и декартова система координат.\circ=4\cdot 3\cdot 1=12$

Ответ: $12$.

Вычисление векторного произведения по координатам векторов

Из определения 1 сразу же вытекает и способ нахождения векторного произведения для двух векторов. Поскольку вектор кроме значения имеет еще и направление, находить его только при помощи скалярной величины невозможно. Но помимо него существует еще способ нахождения с помощью координат данных нам векторов.

Пусть нам даны векторы $\overline{α}$ и $\overline{β}$, которые будут иметь координаты $(α_1,α_2,α_3)$ и $(β_1,β_2,β_3)$, соответственно. Тогда вектор векторного произведения (а именно его координаты) можно найти по следующей формуле:

$\overline{α}х\overline{β}=\begin{vmatrix}\overline{i}&\overline{j}&\overline{k}\\α_1&α_2&α_3\\β_1&β_2&β_3\end{vmatrix}$

Иначе, раскрывая определитель, получим следующие координаты

$\overline{α}х\overline{β}=(α_2 β_3-α_3 β_2,α_3 β_1-α_1 β_3,α_1 β_2-α_2 β_1)$

Пример 2

Найти вектор векторного произведения коллинеарных векторов $\overline{α}$ и $\overline{β}$ с координатами $(0,3,3)$ и $(-1,2,6)$.

Решение .

Воспользуемся формулой, приведенной выше. Получим

$\overline{α}х\overline{β}=\begin{vmatrix}\overline{i}&\overline{j}&\overline{k}\\0&3&3\\-1&2&6\end{vmatrix}=(18-6)\overline{i}-(0+3)\overline{j}+(0+3)\overline{k}=12\overline{i}-3\overline{j}+3\overline{k}=(12,-3,3)$

Ответ: $(12,-3,3)$.

Свойства векторного произведения векторов

Для произвольных смешанных трех векторов $\overline{α}$, $\overline{β}$ и $\overline{γ}$, а также $r∈R$ справедливы следующие свойства:

Пример 3

Найдите площадь параллелограмма, вершины которого имеют координаты $(3,0,0)$, $(0,0,0)$, $(0,8,0)$ и $(3,8,0)$.

Решение .

Вначале изобразим данный параллелограмм в координатном пространстве (рис.5):

Рисунок 5. Параллелограмм в координатном пространстве. Автор24 — интернет-биржа студенческих работ

Видим, что две стороны этого параллелограмма построены с помощью коллинеарных векторов с координатами $\overline{α}=(3,0,0)$ и $\overline{β}=(0,8,0)$.2}=24$

Перед тем, как дать понятие векторного произведения, обратимся к вопросу о ориентации упорядоченной тройки векторов a → , b → , c → в трехмерном пространстве.

Отложим для начала векторы a → , b → , c → от одной точки. Ориентация тройки a → , b → , c → бывает правой или левой, в зависимости от направления самого вектора c → . От того, в какую сторону осуществляется кратчайший поворот от вектора a → к b → с конца вектора c → , будет определен вид тройки a → , b → , c → .

Если кратчайший поворот осуществляется против часовой стрелки, то тройка векторов a → , b → , c → называется правой , если по часовой стрелке – левой .

Далее возьмем два не коллинеарных вектора a → и b → . Отложим затем от точки A векторы A B → = a → и A C → = b → . Построим вектор A D → = c → , который одновременно перпендикулярный одновременно и A B → и A C → . Таким образом, при построении самого вектора A D → = c → мы можем поступить двояко, задав ему либо одно направление, либо противоположное (смотрите иллюстрацию).

Упорядоченная тройка векторов a → , b → , c → может быть, как мы выяснили правой или левой в зависимости от направления вектора.

Из вышесказанного можем ввести определение векторного произведения. Данное определение дается для двух векторов, определенных в прямоугольной системе координат трехмерного пространства.

Определение 1

Векторным произведением двух векторов a → и b → будем называть такой вектор заданный в прямоугольной системе координат трехмерного пространства такой, что:

  • если векторы a → и b → коллинеарны, он будет нулевым;
  • он будет перпендикулярен и вектору a → ​​​​ и вектору b → т.е. ∠ a → c → = ∠ b → c → = π 2 ;
  • его длина определяется по формуле: c → = a → · b → · sin ∠ a → , b → ;
  • тройка векторов a → , b → , c → имеет такую же ориентацию, что и заданная система координат.

Векторное произведение векторов a → и b → имеет следущее обозначение: a → × b → .

Координаты векторного произведения

Так как любой вектор имеет определенные координаты в системе координат, то можно ввести второе определение векторного произведения, которое позволит находить его координаты по заданным координатам векторов.

Определение 2

В прямоугольной системе координат трехмерного пространства векторным произведением двух векторов a → = (a x ; a y ; a z) и b → = (b x ; b y ; b z) называют вектор c → = a → × b → = (a y · b z — a z · b y) · i → + (a z · b x — a x · b z) · j → + (a x · b y — a y · b x) · k → , где i → , j → , k → являются координатными векторами.

Векторное произведение можно представит как определитель квадратной матрицы третьего порядка, где первая строка есть векторы орты i → , j → , k → , вторая строка содержит координаты вектора a → , а третья – координаты вектора b → в заданной прямоугольной системе координат, данный определитель матрицы выглядит так: c → = a → × b → = i → j → k → a x a y a z b x b y b z

Разложив данный определитель по элементам первой строки, получим равенство: c → = a → × b → = i → j → k → a x a y a z b x b y b z = a y a z b y b z · i → — a x a z b x b z · j → + a x a y b x b y · k → = = a → × b → = (a y · b z — a z · b y) · i → + (a z · b x — a x · b z) · j → + (a x · b y — a y · b x) · k →

Свойства векторного произведения

Известно, что векторное произведение в координатах представляется как определитель матрицы c → = a → × b → = i → j → k → a x a y a z b x b y b z , то на базе свойств определителя матрицы выводятся следующие свойства векторного произведения:

  1. антикоммутативность a → × b → = — b → × a → ;
  2. дистрибутивность a (1) → + a (2) → × b = a (1) → × b → + a (2) → × b → или a → × b (1) → + b (2) → = a → × b (1) → + a → × b (2) → ;
  3. ассоциативность λ · a → × b → = λ · a → × b → или a → × (λ · b →) = λ · a → × b → , где λ — произвольное действительное число.

Данные свойства имеют не сложные доказательства.

Для примера можем доказать свойство антикоммутативности векторного произведения.

Доказательство антикоммутативности

По определению a → × b → = i → j → k → a x a y a z b x b y b z и b → × a → = i → j → k → b x b y b z a x a y a z . А если две строчки матрицы переставить местами, то значение определителя матрицы должно меняется на противоположное,следовательно, a → × b → = i → j → k → a x a y a z b x b y b z = — i → j → k → b x b y b z a x a y a z = — b → × a → , что и доказывает антикоммутативность векторного произведения.

Векторное произведение – примеры и решения

В большинстве случаев встречаются три типа задач.

В задачах первого типа обычно заданы длины двух векторов и угол между ними, а нужно найти длину векторного произведения. В этом случае пользуются следующей формулой c → = a → · b → · sin ∠ a → , b → .

Пример 1

Найдите длину векторного произведения векторов a → и b → , если известно a → = 3 , b → = 5 , ∠ a → , b → = π 4 .

Решение

С помощью определения длины векторного произведения векторов a → и b → решим данную задач: a → × b → = a → · b → · sin ∠ a → , b → = 3 · 5 · sin π 4 = 15 2 2 .

Ответ: 15 2 2 .

Задачи второго типа имеют связь с координатами векторов, в них векторное произведение, его длина и т.д. ищутся через известные координаты заданных векторов a → = (a x ; a y ; a z) и b → = (b x ; b y ; b z) .

Для такого типа задач, можно решить массу вариантов заданий. Например, могут быть заданы не координаты векторов a → и b → , а их разложения по координатным векторам вида b → = b x · i → + b y · j → + b z · k → и c → = a → × b → = (a y · b z — a z · b y) · i → + (a z · b x — a x · b z) · j → + (a x · b y — a y · b x) · k → , или векторы a → и b → могут быть заданы координатами точек их начала и конца.

Рассмотрим следующие примеры.

Пример 2

В прямоугольной системе координат заданы два вектора a → = (2 ; 1 ; — 3) , b → = (0 ; — 1 ; 1) . Найдите их векторное произведение.

Решение

По второму определению найдем векторное произведение двух векторов в заданных координатах: a → × b → = (a y · b z — a z · b y) · i → + (a z · b x — a x · b z) · j → + (a x · b y — a y · b x) · k → = = (1 · 1 — (- 3) · (- 1)) · i → + ((- 3) · 0 — 2 · 1) · j → + (2 · (- 1) — 1 · 0) · k → = = — 2 i → — 2 j → — 2 k → .

Если записать векторное произведение через определитель матрицы, то решение данного примера выглядит следующим образом: a → × b → = i → j → k → a x a y a z b x b y b z = i → j → k → 2 1 — 3 0 — 1 1 = — 2 i → — 2 j → — 2 k → .

Ответ: a → × b → = — 2 i → — 2 j → — 2 k → .

Пример 3

Найдите длину векторного произведения векторов i → — j → и i → + j → + k → , где i → , j → , k → — орты прямоугольной декартовой системы координат.

Решение

Для начала найдем координаты заданного векторного произведения i → — j → × i → + j → + k → в данной прямоугольной системе координат.

Известно, что векторы i → — j → и i → + j → + k → имеют координаты (1 ; — 1 ; 0) и (1 ; 1 ; 1) соответственно. Найдем длину векторного произведения при помощи определителя матрицы, тогда имеем i → — j → × i → + j → + k → = i → j → k → 1 — 1 0 1 1 1 = — i → — j → + 2 k → .

Следовательно, векторное произведение i → — j → × i → + j → + k → имеет координаты (- 1 ; — 1 ; 2) в заданной системе координат.

Длину векторного произведения найдем по формуле (см. в разделе нахождение длины вектора): i → — j → × i → + j → + k → = — 1 2 + — 1 2 + 2 2 = 6 .

Ответ: i → — j → × i → + j → + k → = 6 . .

Пример 4

В прямоугольной декартовой системе координат заданы координаты трех точек A (1 , 0 , 1) , B (0 , 2 , 3) , C (1 , 4 , 2) . Найдите какой-нибудь вектор, перпендикулярный A B → и A C → одновременно.

Решение

Векторы A B → и A C → имеют следующие координаты (- 1 ; 2 ; 2) и (0 ; 4 ; 1) соответственно. Найдя векторное произведение векторов A B → и A C → , очевидно, что оно является перпендикулярным вектором по определению и к A B → ​​​​​ и к A C → , то есть, является решением нашей задачи. Найдем его A B → × A C → = i → j → k → — 1 2 2 0 4 1 = — 6 i → + j → — 4 k → .

Ответ: — 6 i → + j → — 4 k → . — один из перпендикулярных векторов.

Задачи третьего типа ориентированы на использование свойств векторного произведения векторов. После применения которых, будем получать решение заданной задачи.

Пример 5

Векторы a → и b → перпендикулярны и их длины равны соответственно 3 и 4 . Найдите длину векторного произведения 3 · a → — b → × a → — 2 · b → = 3 · a → × a → — 2 · b → + — b → × a → — 2 · b → = = 3 · a → × a → + 3 · a → × — 2 · b → + — b → × a → + — b → × — 2 · b → .

Решение

По свойству дистрибутивности векторного произведения мы можем записать 3 · a → — b → × a → — 2 · b → = 3 · a → × a → — 2 · b → + — b → × a → — 2 · b → = = 3 · a → × a → + 3 · a → × — 2 · b → + — b → × a → + — b → × — 2 · b →

По свойству ассоциативности вынесем числовые коэффициенты за знак векторных произведений в последнем выражении: 3 · a → × a → + 3 · a → × — 2 · b → + — b → × a → + — b → × — 2 · b → = = 3 · a → × a → + 3 · (- 2) · a → × b → + (- 1) · b → × a → + (- 1) · (- 2) · b → × b → = = 3 · a → × a → — 6 · a → × b → — b → × a → + 2 · b → × b →

Векторные произведения a → × a → и b → × b → равны 0, так как a → × a → = a → · a → · sin 0 = 0 и b → × b → = b → · b → · sin 0 = 0 , тогда 3 · a → × a → — 6 · a → × b → — b → × a → + 2 · b → × b → = — 6 · a → × b → — b → × a → . .

Из антикоммутативности векторного произведения следует — 6 · a → × b → — b → × a → = — 6 · a → × b → — (- 1) · a → × b → = — 5 · a → × b → . .

Воспользовавшись свойствами векторного произведения, получаем равенство 3 · a → — b → × a → — 2 · b → = = — 5 · a → × b → .

По условию векторы a → и b → перпендикулярны, то есть угол между ними равен π 2 . Теперь остается лишь подставить найденные значения в соответствующие формулы: 3 · a → — b → × a → — 2 · b → = — 5 · a → × b → = = 5 · a → × b → = 5 · a → · b → · sin (a → , b →) = 5 · 3 · 4 · sin π 2 = 60 .

Ответ: 3 · a → — b → × a → — 2 · b → = 60 .

Длина векторного произведения векторов по орпеделению равна a → × b → = a → · b → · sin ∠ a → , b → . Так как уже известно (из школьного курса), что площадь треугольника равна половине произведения длин двух его сторон умноженное на синус угла между данными сторонами. Следовательно, длина векторного произведения равна площади параллелограмма — удвоенного треугольника, а именно произведению сторон в виде векторов a → и b → , отложенные от одной точки, на синус угла между ними sin ∠ a → , b → .

Это и есть геометрический смысл векторного произведения.

Физический смысл векторного произведения

В механике, одном из разделов физики, благодаря векторному произведению можно определить момент силы относительно точки пространства.

Определение 3

Под моментом силы F → , приложенной к точке B , относительно точки A будем понимать следующее векторное произведение A B → × F → .

Если вы заметили ошибку в тексте, пожалуйста, выделите её и нажмите Ctrl+Enter

На данном уроке мы рассмотрим ещё две операции с векторами: векторное произведение векторов и смешанное произведение векторов (сразу ссылка, кому нужно именно оно) . Ничего страшного, так иногда бывает, что для полного счастья, помимо скалярного произведения векторов , требуется ещё и ещё. Такая вот векторная наркомания. Может сложиться впечатление, что мы залезаем в дебри аналитической геометрии. Это не так. В данном разделе высшей математики вообще мало дров, разве что на Буратино хватит. На самом деле материал очень распространенный и простой – вряд ли сложнее, чем то же скалярное произведение , даже типовых задач поменьше будет. Главное в аналитической геометрии, как многие убедятся или уже убедились, НЕ ОШИБАТЬСЯ В ВЫЧИСЛЕНИЯХ. Повторяйте как заклинание, и будет вам счастье =)

Если векторы сверкают где-то далеко, как молнии на горизонте, не беда, начните с урока Векторы для чайников , чтобы восстановить или вновь приобрести базовые знания о векторах. Более подготовленные читатели могут знакомиться с информацией выборочно, я постарался собрать максимально полную коллекцию примеров, которые часто встречаются в практических работах

Чем вас сразу порадовать? Когда я был маленьким, то умел жонглировать двумя и даже тремя шариками. Ловко получалось. Сейчас жонглировать не придётся вообще, поскольку мы будем рассматривать только пространственные векторы , а плоские векторы с двумя координатами останутся за бортом. Почему? Такими уж родились данные действия – векторное и смешанное произведение векторов определены и работают в трёхмерном пространстве. Уже проще!

В данной операции, точно так же, как и в скалярном произведении, участвуют два вектора . Пусть это будут нетленные буквы .

Само действие обозначается следующим образом: . Существуют и другие варианты, но я привык обозначать векторное произведение векторов именно так, в квадратных скобках с крестиком.

И сразу вопрос : если в скалярном произведении векторов участвуют два вектора, и здесь тоже умножаются два вектора, тогда в чём разница ? Явная разница, прежде всего, в РЕЗУЛЬТАТЕ:

Результатом скалярного произведения векторов является ЧИСЛО:

Результатом векторного произведения векторов является ВЕКТОР : , то есть умножаем векторы и получаем снова вектор. Закрытый клуб. Собственно, отсюда и название операции. В различной учебной литературе обозначения тоже могут варьироваться, я буду использовать букву .

Определение векторного произведения

Сначала будет определение с картинкой, затем комментарии.

Определение : Векторным произведением неколлинеарных векторов , взятых в данном порядке , называется ВЕКТОР , длина которого численно равна площади параллелограмма , построенного на данных векторах; вектор ортогонален векторам , и направлен так, что базис имеет правую ориентацию:

Разбираем определение по косточкам, тут много интересного!

Итак, можно выделить следующие существенные моменты:

1) Исходные векторы , обозначенные красными стрелками, по определению не коллинеарны . Случай коллинеарных векторов будет уместно рассмотреть чуть позже.

2) Векторы взяты в строго определённом порядке : – «а» умножается на «бэ» , а не «бэ» на «а». Результатом умножения векторов является ВЕКТОР , который обозначен синим цветом. Если векторы умножить в обратном порядке, то получим равный по длине и противоположный по направлению вектор (малиновый цвет). То есть, справедливо равенство .

3) Теперь познакомимся с геометрическим смыслом векторного произведения. Это очень важный пункт! ДЛИНА синего вектора (а, значит, и малинового вектора ) численно равна ПЛОЩАДИ параллелограмма, построенного на векторах . На рисунке данный параллелограмм заштрихован чёрным цветом.

Примечание : чертёж является схематическим, и, естественно, номинальная длина векторного произведения не равна площади параллелограмма.

Вспоминаем одну из геометрических формул: площадь параллелограмма равна произведению смежных сторон на синус угла между ними . Поэтому, исходя из вышесказанного, справедлива формула вычисления ДЛИНЫ векторного произведения:

Подчёркиваю, что в формуле речь идёт о ДЛИНЕ вектора, а не о самом векторе . Каков практический смысл? А смысл таков, что в задачах аналитической геометрии площадь параллелограмма часто находят через понятие векторного произведения:

Получим вторую важную формулу. Диагональ параллелограмма (красный пунктир) делит его на два равных треугольника. Следовательно, площадь треугольника, построенного на векторах (красная штриховка), можно найти по формуле:

4) Не менее важный факт состоит в том, что вектор ортогонален векторам , то есть . Разумеется, противоположно направленный вектор (малиновая стрелка) тоже ортогонален исходным векторам .

5) Вектор направлен так, что базис имеет правую ориентацию. На уроке о переходе к новому базису я достаточно подробно рассказал об ориентации плоскости , и сейчас мы разберёмся, что такое ориентация пространства. Объяснять буду на пальцах вашей правой руки . Мысленно совместите указательный палец с вектором и средний палец с вектором . Безымянный палец и мизинец прижмите к ладони. В результате большой палец – векторное произведение будет смотреть вверх. Это и есть правоориентированный базис (на рисунке именно он). Теперь поменяйте векторы (указательный и средний пальцы ) местами, в результате большой палец развернётся, и векторное произведение уже будет смотреть вниз. Это тоже правоориентированный базис. Возможно, у вас возник вопрос: а какой базис имеет левую ориентацию? «Присвойте» тем же пальцам левой руки векторы , и полУчите левый базис и левую ориентацию пространства (в этом случае большой палец расположится по направлению нижнего вектора) . Образно говоря, данные базисы «закручивают» или ориентируют пространство в разные стороны. И это понятие не следует считать чем-то надуманным или абстрактным – так, например, ориентацию пространства меняет самое обычное зеркало, и если «вытащить отражённый объект из зазеркалья», то его в общем случае не удастся совместить с «оригиналом». Кстати, поднесите к зеркалу три пальца и проанализируйте отражение;-)

…как всё-таки хорошо, что вы теперь знаете о право- и левоориентированных базисах, ибо страшнЫ высказывания некоторых лекторов о смене ориентации =)

Векторное произведение коллинеарных векторов

Определение подробно разобрано, осталось выяснить, что происходит, когда векторы коллинеарны. Если векторы коллинеарны, то их можно расположить на одной прямой и наш параллелограмм тоже «складывается» в одну прямую. Площадь такого, как говорят математики, вырожденного параллелограмма равна нулю. Это же следует и из формулы – синус нуля или 180-ти градусов равен нулю, а значит, и площадь нулевая

Таким образом, если , то и . Обратите внимание, что само векторное произведение равно нулевому вектору, но на практике этим часто пренебрегают и пишут, что оно тоже равно нулю.

Частный случай – векторное произведение вектора на самого себя:

С помощью векторного произведения можно проверять коллинеарность трёхмерных векторов, и данную задачу среди прочих мы тоже разберём.

Для решения практических примеров может потребоваться тригонометрическая таблица , чтобы находить по ней значения синусов.

Ну что же, разжигаем огонь:

Пример 1

а) Найти длину векторного произведения векторов , если

б) Найти площадь параллелограмма, построенного на векторах , если

Решение : Нет, это не опечатка, исходные данные в пунктах условия я намеренно сделал одинаковыми. Потому что оформление решений будет отличаться!

а) По условию требуется найти длину вектора (векторного произведения). По соответствующей формуле:

Ответ :

Коль скоро спрашивалось о длине, то в ответе указываем размерность – единицы.

б) По условию требуется найти площадь параллелограмма, построенного на векторах . Площадь данного параллелограмма численно равна длине векторного произведения:

Ответ :

Обратите внимание, что в ответе о векторном произведении речи не идёт вообще, нас спрашивали о площади фигуры , соответственно, размерность – квадратные единицы.

Всегда смотрим, ЧТО требуется найти по условию, и, исходя из этого, формулируем чёткий ответ. Может показаться буквоедством, но буквоедов среди преподавателей хватает, и задание с хорошими шансами вернётся на доработку. Хотя это не особо натянутая придирка – если ответ некорректен, то складывается впечатление, что человек не разбирается в простых вещах и/или не вник в суть задания. Этот момент всегда нужно держать на контроле, решая любую задачу по высшей математике, да и по другим предметам тоже.

Куда подевалась большая буковка «эн»? В принципе, её можно было дополнительно прилепить в решение, но в целях сократить запись, я этого не сделал. Надеюсь, всем понятно, что и – это обозначение одного и того же.

Популярный пример для самостоятельного решения:

Пример 2

Найти площадь треугольника, построенного на векторах , если

Формула нахождения площади треугольника через векторное произведение дана в комментариях к определению. Решение и ответ в конце урока.

На практике задача действительно очень распространена, треугольниками вообще могут замучить.

Для решения других задач нам понадобятся:

Свойства векторного произведения векторов

Некоторые свойства векторного произведения мы уже рассмотрели, тем не менее, я их включу в данный список.

Для произвольных векторов и произвольного числа справедливы следующие свойства:

1) В других источниках информации данный пункт обычно не выделяют в свойствах, но он очень важен в практическом плане. Поэтому пусть будет.

2) – свойство тоже разобрано выше, иногда его называют антикоммутативностью . Иными словами, порядок векторов имеет значение.

3) – сочетательные или ассоциативные законы векторного произведения. Константы безпроблемно выносятся за пределы векторного произведения. Действительно, чего им там делать?

4) – распределительные или дистрибутивные законы векторного произведения. С раскрытием скобок тоже нет проблем.

В качестве демонстрации рассмотрим коротенький пример:

Пример 3

Найти , если

Решение: По условию снова требуется найти длину векторного произведения. Распишем нашу миниатюру:

(1) Согласно ассоциативным законам, выносим константы за переделы векторного произведения.

(2) Выносим константу за пределы модуля, при этом модуль «съедает» знак «минус». Длина же не может быть отрицательной.

(3) Дальнейшее понятно.

Ответ :

Пора подбросить дров в огонь:

Пример 4

Вычислить площадь треугольника, построенного на векторах , если

Решение : Площадь треугольника найдём по формуле . Загвоздка состоит в том, что векторы «цэ» и «дэ» сами представлены в виде сумм векторов. Алгоритм здесь стандартен и чем-то напоминает примеры № 3 и 4 урока Скалярное произведение векторов . Решение для ясности разобьём на три этапа:

1) На первом шаге выразим векторное произведение через векторное произведение , по сути, выразим вектор через вектор . О длинах пока ни слова!

(1) Подставляем выражения векторов .

(2) Используя дистрибутивные законы, раскрываем скобки по правилу умножения многочленов.

(3) Используя ассоциативные законы, выносим все константы за пределы векторных произведений. При маломальском опыте действия 2 и 3 можно выполнять одновременно.

(4) Первое и последнее слагаемое равно нулю (нулевому вектору) благодаря приятному свойству . Во втором слагаемом используем свойство антикоммутативности векторного произведения:

(5) Приводим подобные слагаемые.

В результате вектор оказался выражен через вектор, чего и требовалось достичь:

2) На втором шаге найдем длину нужного нам векторного произведения. Данное действие напоминает Пример 3:

3) Найдём площадь искомого треугольника:

Этапы 2-3 решения можно было оформить и одной строкой.

Ответ :

Рассмотренная задача достаточно распространена в контрольных работах, вот пример для самостоятельного решения:

Пример 5

Найти , если

Краткое решение и ответ в конце урока. Посмотрим, насколько вы были внимательны при изучении предыдущих примеров;-)

Векторное произведение векторов в координатах , заданных в ортонормированном базисе , выражается формулой :

Формула и правда простецкая: в верхнюю строку определителя записываем координатные векторы, во вторую и третью строки «укладываем» координаты векторов , причём укладываем в строгом порядке – сначала координаты вектора «вэ», затем координаты вектора «дубль-вэ». Если векторы нужно умножить в другом порядке, то и строки следует поменять местами:

Пример 10

Проверить, будут ли коллинеарны следующие векторы пространства:
а)
б)

Решение : Проверка основана на одном из утверждений данного урока: если векторы коллинеарны, то их векторное произведение равно нулю (нулевому вектору): .

а) Найдём векторное произведение:

Таким образом, векторы не коллинеарны.

б) Найдём векторное произведение:

Ответ : а) не коллинеарны, б)

Вот, пожалуй, и все основные сведения о векторном произведении векторов.

Данный раздел будет не очень большим, так как задач, где используется смешанное произведение векторов, немного. Фактически всё будет упираться в определение, геометрический смысл и пару рабочих формул.

Смешанное произведение векторов – это произведение трёх векторов :

Вот так вот они выстроились паровозиком и ждут, не дождутся, когда их вычислят.

Сначала опять определение и картинка:

Определение : Смешанным произведением некомпланарных векторов , взятых в данном порядке , называется объём параллелепипеда , построенного на данных векторах, снабжённый знаком «+», если базис правый, и знаком «–», если базис левый.

Выполним рисунок. Невидимые нам линии прочерчены пунктиром:

Погружаемся в определение:

2) Векторы взяты в определённом порядке , то есть перестановка векторов в произведении , как вы догадываетесь, не проходит без последствий.

3) Перед тем, как прокомментировать геометрический смысл, отмечу очевидный факт: смешанное произведение векторов является ЧИСЛОМ : . В учебной литературе оформление может быть несколько другим, я привык обозначать смешанное произведение через , а результат вычислений буквой «пэ».

По определению смешанное произведение – это объем параллелепипеда , построенного на векторах (фигура прочерчена красными векторами и линиями чёрного цвета). То есть, число равно объему данного параллелепипеда.

Примечание : чертёж является схематическим.

4) Не будем заново париться с понятием ориентации базиса и пространства. Смысл заключительной части состоит в том, что к объёму может добавляться знак минус. Простыми словами, смешанное произведение может быть отрицательным: .

Непосредственно из определения следует формула вычисления объема параллелепипеда, построенного на векторах .

Свойства произведения векторов. Векторное произведение векторов онлайн. Векторное произведение – примеры и решения

Перед тем, как дать понятие векторного произведения, обратимся к вопросу о ориентации упорядоченной тройки векторов a → , b → , c → в трехмерном пространстве.

Отложим для начала векторы a → , b → , c → от одной точки. Ориентация тройки a → , b → , c → бывает правой или левой, в зависимости от направления самого вектора c → . От того, в какую сторону осуществляется кратчайший поворот от вектора a → к b → с конца вектора c → , будет определен вид тройки a → , b → , c → .

Если кратчайший поворот осуществляется против часовой стрелки, то тройка векторов a → , b → , c → называется правой , если по часовой стрелке – левой .

Далее возьмем два не коллинеарных вектора a → и b → . Отложим затем от точки A векторы A B → = a → и A C → = b → . Построим вектор A D → = c → , который одновременно перпендикулярный одновременно и A B → и A C → . Таким образом, при построении самого вектора A D → = c → мы можем поступить двояко, задав ему либо одно направление, либо противоположное (смотрите иллюстрацию).

Упорядоченная тройка векторов a → , b → , c → может быть, как мы выяснили правой или левой в зависимости от направления вектора.

Из вышесказанного можем ввести определение векторного произведения. Данное определение дается для двух векторов, определенных в прямоугольной системе координат трехмерного пространства.

Определение 1

Векторным произведением двух векторов a → и b → будем называть такой вектор заданный в прямоугольной системе координат трехмерного пространства такой, что:

  • если векторы a → и b → коллинеарны, он будет нулевым;
  • он будет перпендикулярен и вектору a → ​​​​ и вектору b → т.е. ∠ a → c → = ∠ b → c → = π 2 ;
  • его длина определяется по формуле: c → = a → · b → · sin ∠ a → , b → ;
  • тройка векторов a → , b → , c → имеет такую же ориентацию, что и заданная система координат.

Векторное произведение векторов a → и b → имеет следущее обозначение: a → × b → .

Координаты векторного произведения

Так как любой вектор имеет определенные координаты в системе координат, то можно ввести второе определение векторного произведения, которое позволит находить его координаты по заданным координатам векторов.

Определение 2

В прямоугольной системе координат трехмерного пространства векторным произведением двух векторов a → = (a x ; a y ; a z) и b → = (b x ; b y ; b z) называют вектор c → = a → × b → = (a y · b z — a z · b y) · i → + (a z · b x — a x · b z) · j → + (a x · b y — a y · b x) · k → , где i → , j → , k → являются координатными векторами.

Векторное произведение можно представит как определитель квадратной матрицы третьего порядка, где первая строка есть векторы орты i → , j → , k → , вторая строка содержит координаты вектора a → , а третья – координаты вектора b → в заданной прямоугольной системе координат, данный определитель матрицы выглядит так: c → = a → × b → = i → j → k → a x a y a z b x b y b z

Разложив данный определитель по элементам первой строки, получим равенство: c → = a → × b → = i → j → k → a x a y a z b x b y b z = a y a z b y b z · i → — a x a z b x b z · j → + a x a y b x b y · k → = = a → × b → = (a y · b z — a z · b y) · i → + (a z · b x — a x · b z) · j → + (a x · b y — a y · b x) · k →

Свойства векторного произведения

Известно, что векторное произведение в координатах представляется как определитель матрицы c → = a → × b → = i → j → k → a x a y a z b x b y b z , то на базе свойств определителя матрицы выводятся следующие свойства векторного произведения:

  1. антикоммутативность a → × b → = — b → × a → ;
  2. дистрибутивность a (1) → + a (2) → × b = a (1) → × b → + a (2) → × b → или a → × b (1) → + b (2) → = a → × b (1) → + a → × b (2) → ;
  3. ассоциативность λ · a → × b → = λ · a → × b → или a → × (λ · b →) = λ · a → × b → , где λ — произвольное действительное число.

Данные свойства имеют не сложные доказательства.

Для примера можем доказать свойство антикоммутативности векторного произведения.

Доказательство антикоммутативности

По определению a → × b → = i → j → k → a x a y a z b x b y b z и b → × a → = i → j → k → b x b y b z a x a y a z . А если две строчки матрицы переставить местами, то значение определителя матрицы должно меняется на противоположное,следовательно, a → × b → = i → j → k → a x a y a z b x b y b z = — i → j → k → b x b y b z a x a y a z = — b → × a → , что и доказывает антикоммутативность векторного произведения.

Векторное произведение – примеры и решения

В большинстве случаев встречаются три типа задач.

В задачах первого типа обычно заданы длины двух векторов и угол между ними, а нужно найти длину векторного произведения. В этом случае пользуются следующей формулой c → = a → · b → · sin ∠ a → , b → .

Пример 1

Найдите длину векторного произведения векторов a → и b → , если известно a → = 3 , b → = 5 , ∠ a → , b → = π 4 .

Решение

С помощью определения длины векторного произведения векторов a → и b → решим данную задач: a → × b → = a → · b → · sin ∠ a → , b → = 3 · 5 · sin π 4 = 15 2 2 .

Ответ: 15 2 2 .

Задачи второго типа имеют связь с координатами векторов, в них векторное произведение, его длина и т.д. ищутся через известные координаты заданных векторов a → = (a x ; a y ; a z) и b → = (b x ; b y ; b z) .

Для такого типа задач, можно решить массу вариантов заданий. Например, могут быть заданы не координаты векторов a → и b → , а их разложения по координатным векторам вида b → = b x · i → + b y · j → + b z · k → и c → = a → × b → = (a y · b z — a z · b y) · i → + (a z · b x — a x · b z) · j → + (a x · b y — a y · b x) · k → , или векторы a → и b → могут быть заданы координатами точек их начала и конца.

Рассмотрим следующие примеры.

Пример 2

В прямоугольной системе координат заданы два вектора a → = (2 ; 1 ; — 3) , b → = (0 ; — 1 ; 1) . Найдите их векторное произведение.

Решение

По второму определению найдем векторное произведение двух векторов в заданных координатах: a → × b → = (a y · b z — a z · b y) · i → + (a z · b x — a x · b z) · j → + (a x · b y — a y · b x) · k → = = (1 · 1 — (- 3) · (- 1)) · i → + ((- 3) · 0 — 2 · 1) · j → + (2 · (- 1) — 1 · 0) · k → = = — 2 i → — 2 j → — 2 k → .

Если записать векторное произведение через определитель матрицы, то решение данного примера выглядит следующим образом: a → × b → = i → j → k → a x a y a z b x b y b z = i → j → k → 2 1 — 3 0 — 1 1 = — 2 i → — 2 j → — 2 k → .

Ответ: a → × b → = — 2 i → — 2 j → — 2 k → .

Пример 3

Найдите длину векторного произведения векторов i → — j → и i → + j → + k → , где i → , j → , k → — орты прямоугольной декартовой системы координат.

Решение

Для начала найдем координаты заданного векторного произведения i → — j → × i → + j → + k → в данной прямоугольной системе координат.

Известно, что векторы i → — j → и i → + j → + k → имеют координаты (1 ; — 1 ; 0) и (1 ; 1 ; 1) соответственно. Найдем длину векторного произведения при помощи определителя матрицы, тогда имеем i → — j → × i → + j → + k → = i → j → k → 1 — 1 0 1 1 1 = — i → — j → + 2 k → .

Следовательно, векторное произведение i → — j → × i → + j → + k → имеет координаты (- 1 ; — 1 ; 2) в заданной системе координат.

Длину векторного произведения найдем по формуле (см. в разделе нахождение длины вектора): i → — j → × i → + j → + k → = — 1 2 + — 1 2 + 2 2 = 6 .

Ответ: i → — j → × i → + j → + k → = 6 . .

Пример 4

В прямоугольной декартовой системе координат заданы координаты трех точек A (1 , 0 , 1) , B (0 , 2 , 3) , C (1 , 4 , 2) . Найдите какой-нибудь вектор, перпендикулярный A B → и A C → одновременно.

Решение

Векторы A B → и A C → имеют следующие координаты (- 1 ; 2 ; 2) и (0 ; 4 ; 1) соответственно. Найдя векторное произведение векторов A B → и A C → , очевидно, что оно является перпендикулярным вектором по определению и к A B → ​​​​​ и к A C → , то есть, является решением нашей задачи. Найдем его A B → × A C → = i → j → k → — 1 2 2 0 4 1 = — 6 i → + j → — 4 k → .

Ответ: — 6 i → + j → — 4 k → . — один из перпендикулярных векторов.

Задачи третьего типа ориентированы на использование свойств векторного произведения векторов. После применения которых, будем получать решение заданной задачи.

Пример 5

Векторы a → и b → перпендикулярны и их длины равны соответственно 3 и 4 . Найдите длину векторного произведения 3 · a → — b → × a → — 2 · b → = 3 · a → × a → — 2 · b → + — b → × a → — 2 · b → = = 3 · a → × a → + 3 · a → × — 2 · b → + — b → × a → + — b → × — 2 · b → .

Решение

По свойству дистрибутивности векторного произведения мы можем записать 3 · a → — b → × a → — 2 · b → = 3 · a → × a → — 2 · b → + — b → × a → — 2 · b → = = 3 · a → × a → + 3 · a → × — 2 · b → + — b → × a → + — b → × — 2 · b →

По свойству ассоциативности вынесем числовые коэффициенты за знак векторных произведений в последнем выражении: 3 · a → × a → + 3 · a → × — 2 · b → + — b → × a → + — b → × — 2 · b → = = 3 · a → × a → + 3 · (- 2) · a → × b → + (- 1) · b → × a → + (- 1) · (- 2) · b → × b → = = 3 · a → × a → — 6 · a → × b → — b → × a → + 2 · b → × b →

Векторные произведения a → × a → и b → × b → равны 0, так как a → × a → = a → · a → · sin 0 = 0 и b → × b → = b → · b → · sin 0 = 0 , тогда 3 · a → × a → — 6 · a → × b → — b → × a → + 2 · b → × b → = — 6 · a → × b → — b → × a → . .

Из антикоммутативности векторного произведения следует — 6 · a → × b → — b → × a → = — 6 · a → × b → — (- 1) · a → × b → = — 5 · a → × b → . .

Воспользовавшись свойствами векторного произведения, получаем равенство 3 · a → — b → × a → — 2 · b → = = — 5 · a → × b → .

По условию векторы a → и b → перпендикулярны, то есть угол между ними равен π 2 . Теперь остается лишь подставить найденные значения в соответствующие формулы: 3 · a → — b → × a → — 2 · b → = — 5 · a → × b → = = 5 · a → × b → = 5 · a → · b → · sin (a → , b →) = 5 · 3 · 4 · sin π 2 = 60 .

Ответ: 3 · a → — b → × a → — 2 · b → = 60 .

Длина векторного произведения векторов по орпеделению равна a → × b → = a → · b → · sin ∠ a → , b → . Так как уже известно (из школьного курса), что площадь треугольника равна половине произведения длин двух его сторон умноженное на синус угла между данными сторонами. Следовательно, длина векторного произведения равна площади параллелограмма — удвоенного треугольника, а именно произведению сторон в виде векторов a → и b → , отложенные от одной точки, на синус угла между ними sin ∠ a → , b → .

Это и есть геометрический смысл векторного произведения.

Физический смысл векторного произведения

В механике, одном из разделов физики, благодаря векторному произведению можно определить момент силы относительно точки пространства.

Определение 3

Под моментом силы F → , приложенной к точке B , относительно точки A будем понимать следующее векторное произведение A B → × F → .

Если вы заметили ошибку в тексте, пожалуйста, выделите её и нажмите Ctrl+Enter

На данном уроке мы рассмотрим ещё две операции с векторами: векторное произведение векторов и смешанное произведение векторов (сразу ссылка, кому нужно именно оно) . Ничего страшного, так иногда бывает, что для полного счастья, помимо скалярного произведения векторов , требуется ещё и ещё. Такая вот векторная наркомания. Может сложиться впечатление, что мы залезаем в дебри аналитической геометрии. Это не так. В данном разделе высшей математики вообще мало дров, разве что на Буратино хватит. На самом деле материал очень распространенный и простой – вряд ли сложнее, чем то же скалярное произведение , даже типовых задач поменьше будет. Главное в аналитической геометрии, как многие убедятся или уже убедились, НЕ ОШИБАТЬСЯ В ВЫЧИСЛЕНИЯХ. Повторяйте как заклинание, и будет вам счастье =)

Если векторы сверкают где-то далеко, как молнии на горизонте, не беда, начните с урока Векторы для чайников , чтобы восстановить или вновь приобрести базовые знания о векторах. Более подготовленные читатели могут знакомиться с информацией выборочно, я постарался собрать максимально полную коллекцию примеров, которые часто встречаются в практических работах

Чем вас сразу порадовать? Когда я был маленьким, то умел жонглировать двумя и даже тремя шариками. Ловко получалось. Сейчас жонглировать не придётся вообще, поскольку мы будем рассматривать только пространственные векторы , а плоские векторы с двумя координатами останутся за бортом. Почему? Такими уж родились данные действия – векторное и смешанное произведение векторов определены и работают в трёхмерном пространстве. Уже проще!

В данной операции, точно так же, как и в скалярном произведении, участвуют два вектора . Пусть это будут нетленные буквы .

Само действие обозначается следующим образом: . Существуют и другие варианты, но я привык обозначать векторное произведение векторов именно так, в квадратных скобках с крестиком.

И сразу вопрос : если в скалярном произведении векторов участвуют два вектора, и здесь тоже умножаются два вектора, тогда в чём разница ? Явная разница, прежде всего, в РЕЗУЛЬТАТЕ:

Результатом скалярного произведения векторов является ЧИСЛО:

Результатом векторного произведения векторов является ВЕКТОР : , то есть умножаем векторы и получаем снова вектор. Закрытый клуб. Собственно, отсюда и название операции. В различной учебной литературе обозначения тоже могут варьироваться, я буду использовать букву .

Определение векторного произведения

Сначала будет определение с картинкой, затем комментарии.

Определение : Векторным произведением неколлинеарных векторов , взятых в данном порядке , называется ВЕКТОР , длина которого численно равна площади параллелограмма , построенного на данных векторах; вектор ортогонален векторам , и направлен так, что базис имеет правую ориентацию:

Разбираем определение по косточкам, тут много интересного!

Итак, можно выделить следующие существенные моменты:

1) Исходные векторы , обозначенные красными стрелками, по определению не коллинеарны . Случай коллинеарных векторов будет уместно рассмотреть чуть позже.

2) Векторы взяты в строго определённом порядке : – «а» умножается на «бэ» , а не «бэ» на «а». Результатом умножения векторов является ВЕКТОР , который обозначен синим цветом. Если векторы умножить в обратном порядке, то получим равный по длине и противоположный по направлению вектор (малиновый цвет). То есть, справедливо равенство .

3) Теперь познакомимся с геометрическим смыслом векторного произведения. Это очень важный пункт! ДЛИНА синего вектора (а, значит, и малинового вектора ) численно равна ПЛОЩАДИ параллелограмма, построенного на векторах . На рисунке данный параллелограмм заштрихован чёрным цветом.

Примечание : чертёж является схематическим, и, естественно, номинальная длина векторного произведения не равна площади параллелограмма.

Вспоминаем одну из геометрических формул: площадь параллелограмма равна произведению смежных сторон на синус угла между ними . Поэтому, исходя из вышесказанного, справедлива формула вычисления ДЛИНЫ векторного произведения:

Подчёркиваю, что в формуле речь идёт о ДЛИНЕ вектора, а не о самом векторе . Каков практический смысл? А смысл таков, что в задачах аналитической геометрии площадь параллелограмма часто находят через понятие векторного произведения:

Получим вторую важную формулу. Диагональ параллелограмма (красный пунктир) делит его на два равных треугольника. Следовательно, площадь треугольника, построенного на векторах (красная штриховка), можно найти по формуле:

4) Не менее важный факт состоит в том, что вектор ортогонален векторам , то есть . Разумеется, противоположно направленный вектор (малиновая стрелка) тоже ортогонален исходным векторам .

5) Вектор направлен так, что базис имеет правую ориентацию. На уроке о переходе к новому базису я достаточно подробно рассказал об ориентации плоскости , и сейчас мы разберёмся, что такое ориентация пространства. Объяснять буду на пальцах вашей правой руки . Мысленно совместите указательный палец с вектором и средний палец с вектором . Безымянный палец и мизинец прижмите к ладони. В результате большой палец – векторное произведение будет смотреть вверх. Это и есть правоориентированный базис (на рисунке именно он). Теперь поменяйте векторы (указательный и средний пальцы ) местами, в результате большой палец развернётся, и векторное произведение уже будет смотреть вниз. Это тоже правоориентированный базис. Возможно, у вас возник вопрос: а какой базис имеет левую ориентацию? «Присвойте» тем же пальцам левой руки векторы , и полУчите левый базис и левую ориентацию пространства (в этом случае большой палец расположится по направлению нижнего вектора) . Образно говоря, данные базисы «закручивают» или ориентируют пространство в разные стороны. И это понятие не следует считать чем-то надуманным или абстрактным – так, например, ориентацию пространства меняет самое обычное зеркало, и если «вытащить отражённый объект из зазеркалья», то его в общем случае не удастся совместить с «оригиналом». Кстати, поднесите к зеркалу три пальца и проанализируйте отражение;-)

…как всё-таки хорошо, что вы теперь знаете о право- и левоориентированных базисах, ибо страшнЫ высказывания некоторых лекторов о смене ориентации =)

Векторное произведение коллинеарных векторов

Определение подробно разобрано, осталось выяснить, что происходит, когда векторы коллинеарны. Если векторы коллинеарны, то их можно расположить на одной прямой и наш параллелограмм тоже «складывается» в одну прямую. Площадь такого, как говорят математики, вырожденного параллелограмма равна нулю. Это же следует и из формулы – синус нуля или 180-ти градусов равен нулю, а значит, и площадь нулевая

Таким образом, если , то и . Обратите внимание, что само векторное произведение равно нулевому вектору, но на практике этим часто пренебрегают и пишут, что оно тоже равно нулю.

Частный случай – векторное произведение вектора на самого себя:

С помощью векторного произведения можно проверять коллинеарность трёхмерных векторов, и данную задачу среди прочих мы тоже разберём.

Для решения практических примеров может потребоваться тригонометрическая таблица , чтобы находить по ней значения синусов.

Ну что же, разжигаем огонь:

Пример 1

а) Найти длину векторного произведения векторов , если

б) Найти площадь параллелограмма, построенного на векторах , если

Решение : Нет, это не опечатка, исходные данные в пунктах условия я намеренно сделал одинаковыми. Потому что оформление решений будет отличаться!

а) По условию требуется найти длину вектора (векторного произведения). По соответствующей формуле:

Ответ :

Коль скоро спрашивалось о длине, то в ответе указываем размерность – единицы.

б) По условию требуется найти площадь параллелограмма, построенного на векторах . Площадь данного параллелограмма численно равна длине векторного произведения:

Ответ :

Обратите внимание, что в ответе о векторном произведении речи не идёт вообще, нас спрашивали о площади фигуры , соответственно, размерность – квадратные единицы.

Всегда смотрим, ЧТО требуется найти по условию, и, исходя из этого, формулируем чёткий ответ. Может показаться буквоедством, но буквоедов среди преподавателей хватает, и задание с хорошими шансами вернётся на доработку. Хотя это не особо натянутая придирка – если ответ некорректен, то складывается впечатление, что человек не разбирается в простых вещах и/или не вник в суть задания. Этот момент всегда нужно держать на контроле, решая любую задачу по высшей математике, да и по другим предметам тоже.

Куда подевалась большая буковка «эн»? В принципе, её можно было дополнительно прилепить в решение, но в целях сократить запись, я этого не сделал. Надеюсь, всем понятно, что и – это обозначение одного и того же.

Популярный пример для самостоятельного решения:

Пример 2

Найти площадь треугольника, построенного на векторах , если

Формула нахождения площади треугольника через векторное произведение дана в комментариях к определению. Решение и ответ в конце урока.

На практике задача действительно очень распространена, треугольниками вообще могут замучить.

Для решения других задач нам понадобятся:

Свойства векторного произведения векторов

Некоторые свойства векторного произведения мы уже рассмотрели, тем не менее, я их включу в данный список.

Для произвольных векторов и произвольного числа справедливы следующие свойства:

1) В других источниках информации данный пункт обычно не выделяют в свойствах, но он очень важен в практическом плане. Поэтому пусть будет.

2) – свойство тоже разобрано выше, иногда его называют антикоммутативностью . Иными словами, порядок векторов имеет значение.

3) – сочетательные или ассоциативные законы векторного произведения. Константы безпроблемно выносятся за пределы векторного произведения. Действительно, чего им там делать?

4) – распределительные или дистрибутивные законы векторного произведения. С раскрытием скобок тоже нет проблем.

В качестве демонстрации рассмотрим коротенький пример:

Пример 3

Найти , если

Решение: По условию снова требуется найти длину векторного произведения. Распишем нашу миниатюру:

(1) Согласно ассоциативным законам, выносим константы за переделы векторного произведения.

(2) Выносим константу за пределы модуля, при этом модуль «съедает» знак «минус». Длина же не может быть отрицательной.

(3) Дальнейшее понятно.

Ответ :

Пора подбросить дров в огонь:

Пример 4

Вычислить площадь треугольника, построенного на векторах , если

Решение : Площадь треугольника найдём по формуле . Загвоздка состоит в том, что векторы «цэ» и «дэ» сами представлены в виде сумм векторов. Алгоритм здесь стандартен и чем-то напоминает примеры № 3 и 4 урока Скалярное произведение векторов . Решение для ясности разобьём на три этапа:

1) На первом шаге выразим векторное произведение через векторное произведение , по сути, выразим вектор через вектор . О длинах пока ни слова!

(1) Подставляем выражения векторов .

(2) Используя дистрибутивные законы, раскрываем скобки по правилу умножения многочленов.

(3) Используя ассоциативные законы, выносим все константы за пределы векторных произведений. При маломальском опыте действия 2 и 3 можно выполнять одновременно.

(4) Первое и последнее слагаемое равно нулю (нулевому вектору) благодаря приятному свойству . Во втором слагаемом используем свойство антикоммутативности векторного произведения:

(5) Приводим подобные слагаемые.

В результате вектор оказался выражен через вектор, чего и требовалось достичь:

2) На втором шаге найдем длину нужного нам векторного произведения. Данное действие напоминает Пример 3:

3) Найдём площадь искомого треугольника:

Этапы 2-3 решения можно было оформить и одной строкой.

Ответ :

Рассмотренная задача достаточно распространена в контрольных работах, вот пример для самостоятельного решения:

Пример 5

Найти , если

Краткое решение и ответ в конце урока. Посмотрим, насколько вы были внимательны при изучении предыдущих примеров;-)

Векторное произведение векторов в координатах , заданных в ортонормированном базисе , выражается формулой :

Формула и правда простецкая: в верхнюю строку определителя записываем координатные векторы, во вторую и третью строки «укладываем» координаты векторов , причём укладываем в строгом порядке – сначала координаты вектора «вэ», затем координаты вектора «дубль-вэ». Если векторы нужно умножить в другом порядке, то и строки следует поменять местами:

Пример 10

Проверить, будут ли коллинеарны следующие векторы пространства:
а)
б)

Решение : Проверка основана на одном из утверждений данного урока: если векторы коллинеарны, то их векторное произведение равно нулю (нулевому вектору): .

а) Найдём векторное произведение:

Таким образом, векторы не коллинеарны.

б) Найдём векторное произведение:

Ответ : а) не коллинеарны, б)

Вот, пожалуй, и все основные сведения о векторном произведении векторов.

Данный раздел будет не очень большим, так как задач, где используется смешанное произведение векторов, немного. Фактически всё будет упираться в определение, геометрический смысл и пару рабочих формул.

Смешанное произведение векторов – это произведение трёх векторов :

Вот так вот они выстроились паровозиком и ждут, не дождутся, когда их вычислят.

Сначала опять определение и картинка:

Определение : Смешанным произведением некомпланарных векторов , взятых в данном порядке , называется объём параллелепипеда , построенного на данных векторах, снабжённый знаком «+», если базис правый, и знаком «–», если базис левый.

Выполним рисунок. Невидимые нам линии прочерчены пунктиром:

Погружаемся в определение:

2) Векторы взяты в определённом порядке , то есть перестановка векторов в произведении , как вы догадываетесь, не проходит без последствий.

3) Перед тем, как прокомментировать геометрический смысл, отмечу очевидный факт: смешанное произведение векторов является ЧИСЛОМ : . В учебной литературе оформление может быть несколько другим, я привык обозначать смешанное произведение через , а результат вычислений буквой «пэ».

По определению смешанное произведение – это объем параллелепипеда , построенного на векторах (фигура прочерчена красными векторами и линиями чёрного цвета). То есть, число равно объему данного параллелепипеда.

Примечание : чертёж является схематическим.

4) Не будем заново париться с понятием ориентации базиса и пространства. Смысл заключительной части состоит в том, что к объёму может добавляться знак минус. Простыми словами, смешанное произведение может быть отрицательным: .

Непосредственно из определения следует формула вычисления объема параллелепипеда, построенного на векторах .

Мы будем использовать таблицу векторного произведения векторов i,j иk:

если направление кратчайшего пути от первого вектора к второму совпадает с направлением стрелки, то произведение равно третьему вектору, если не совпадает — третий вектор берется со знаком «минус».

Пусть заданы два вектора а=ахi +ayj +azk и b =bxi +byj +bzk . Найдем векторное произведение этих векторов, перемножая их как многочлены (согласно свойств векторного произведения):
Полученную формулу можно записать еще короче:так как правая часть равенства (7.1) соответствует разложению определителя третьего порядка по элементам первой строки.Равенство (7.2) легко запоминается.

7.4. Некоторые приложения векторного произведения

Установление коллинеарности векторов.
Нахождение площади параллелограмма и треугольника

Согласно определению векторного произведения векторов а и b |а хb | = |а| * |b |sing , т. е. S пар = |а х b |. И, значит, DS =1/2|а х b |.

Определение момента силы относительно точки

Пусть в точке А приложена сила F =АВ и пусть О — некоторая точка пространства Из физики известно, что моментом си лы F относительно точки О называется вектор М, который проходит через точку О и:

1) перпендикулярен плоскости, проходящей через точки О, А, В;

2) численно равен произведению силы на плечо 3) образует правую тройку с векторами ОА и A В.

Стало быть, М=ОА х F . Нахождение линейной скорости вращения

Скорость v точки М твердого тела, вращающегося с угловой скоростью w вокруг неподвижной оси, определяется формулой Эйлера v =w хr , где r =ОМ, где О-некоторая неподвижная точка оси (см. рис. 21).

Угол между векторами

Из определения скалярного произведения двух векторов следует, что Если векторы и заданы координатами и , то формула (1.6.3.1) запишется в виде:

Площадь параллелограмма,построенных на векторах

Задачи на измерение длин отрезков, расстояний между точками, площадей поверхностей и объемов тел относятся к важному классу проблем, которые принято называть метрическими. В предыдущем разделе мы познакомились с тем, как использовать векторную алгебру для вычисления длин отрезков и расстояний между точками. Теперь мы собираемся найти способы вычисления площадей и объемов. Векторная алгебра позволяет ставить и решать подобные задачи только для достаточно простых случаев. Для вычисления площадей произвольных поверхностей и объемов произвольных тел требуются методы анализа. Но методы анализа в свою очередь существенным образом опираются на те результаты, которые дает векторная алгебра.

Для решения поставленной задачи, мы избрали достаточно долгий и непростой путь, подсказанный Гильбертом Стренгом , связанный с многочисленными геометрическими преобразованиями и кропотливыми алгебраическими вычислениями. Мы избрали этот путь несмотря на то, что существуют другие подходы, которые быстрее приводят к цели потому, что он показался нам прямым и естественным. Прямой путь в науке не всегда оказывается самым простым. Люди искушенные знают об этом и предпочитают пути окольные, но если не попытаться пройти прямиком, то можно так и остаться в неведении относительно некоторых тонкостей теории.

На избранном нами пути естественным образом появляются такие понятия как ориентация пространства, определитель, векторное и смешанное произведения. Особенно наглядно, как под микроскопом, проявляется геометрический смысл определителя и его свойств. Традиционно понятие определителя вводится в теории систем линейных уравнений, но именно для решения таких систем определитель почти бесполезен. Геометрический же смысл определителя существенен для векторной и тензорной алгебры.

А теперь запасемся терпением и начнем с самых простых и понятных случаев.

1. Векторы ориентированы вдоль координатных осей декартовой системы координат.

Пусть вектор a направлен по оси x, а вектор b вдоль оси y. На рис. 21 показаны четыре различных варианта расположения векторов по отношению к осям координат.

Векторы a и b в координатной форме:Где a и b означают модуль соответствующего вектора, а – знак координаты вектора.

Поскольку векторы ортогональны, то параллелограммы, построенные на них, являются прямоугольниками. Их площади равны просто произведению их сторон. Выразим эти произведения через координаты векторов для всех четырех случаев.

Все четыре формулы для вычисления площади одинаковы за исключением знака. Можно было бы просто закрыть на это глаза и записать, что во всех случаях. Однако более продуктивной оказывается другая возможность: придать знаку какой-то смысл. Посмотрим внимательно на рис. 21. В тех случаях, когда, поворот вектора к вектору осуществляется по часовой стрелке. В тех же случаях, когда мы вынуждены использовать в формуле знак минус, поворот вектора к вектору осуществляется против часовой стрелки. Это наблюдение позволяет связать знак в выражениях для площади с ориентацией плоскости.

Площадь прямоугольника, построенного на векторах aиb, со знаком плюс или минус будем считать ориентированной площадью, при этом знак будем связывать с ориентацией, задаваемой векторами. Для ориентированной площади мы можем записать единую формулу для всех рассмотренных четырех случаев:. Знак «векторной» черты над буквой S вводится для того, чтобы отличить обычную площадь, которая всегда положительна, от ориентированной.

При этом, очевидно, что те же самые векторы, взятые в другом порядке, определяют противоположную ориентацию, поэтому, . Просто площадь будем по-прежнему обозначать буквой S и, следовательно, .

Теперь, когда казалось бы ценой расширения понятия площади, мы получили общее выражение, внимательный читатель скажет, что мы рассмотрели не все возможности. Действительно, кроме четырех вариантов расположения векторов, представленных на рис. 21, имеются еще четыре (рис. 22) Запишем снова векторы и в координатной форме: Выразим площади через координаты векторов. 4. . Знаки в новых выражениях не поменялись, но, к сожалению, поменялась ориентация по отношению к предыдущим четырем случаям. Поэтому для ориентированной площади мы вынуждены записать: . Хотя надежда на гениальную простоту и не оправдалась, но, тем не менее, мы все-таки можем записать общее выражение для всех четырех случаев.

То есть, ориентированная площадь прямоугольника, построенного на векторах, как на сторонах, равна определителю, составленному из координат векторов, как из столбцов.

Мы полагаем, что с теорией определителей читатель знаком, поэтому, мы не останавливаемся подробно на этом понятии. Тем не менее, мы даем соответствующие определения, для того чтобы изменить акценты и показать, что к этому понятию можно прийти из чисто геометрических соображений.Итак, , , , – различные формы обозначения для одного и того же понятия – определителя, составленного из координат векторов, как из столбцов. Равенство может быть принято за его определение для двухмерного случая.

2. Вектор b не параллелен оси x; вектор a/ является произвольным вектором.

Для того чтобы свести этот случай к уже известным, рассмотрим некоторые геометрические преобразования параллелограмма, построенного на векторах и (рис. .смешанные произведения векторов и его свойства

Угол между векторами

Для того чтобы мы могли ввести понятие векторного произведения двух векторов, нужно сначала разобраться с таким понятие, как угол между этими векторами.

Пусть нам даны два вектора $\overline{α}$ и $\overline{β}$. Возьмем в пространстве какую-либо точку $O$ и отложим от нее векторы $\overline{α}=\overline{OA}$ и $\overline{β}=\overline{OB}$, тогда угол $AOB$ будет называться углом между этими векторами (рис. 1).

Обозначение: $∠(\overline{α},\overline{β})$

Понятие векторного произведения векторов и формула нахождения

Определение 1

Векторным произведением двух векторов называется вектор, перпендикулярный обоим данным векторам, и его длина будет равняться произведению длин этих векторов с синусом угла между данными векторами, а также этот вектор с двумя начальными имеют туже ориентацию, как и декартова система координат.

Обозначение: $\overline{α}х\overline{β}$.

Математически это выглядит следующим образом:

  1. $|\overline{α}х\overline{β}|=|\overline{α}||\overline{β}|sin⁡∠(\overline{α},\overline{β})$
  2. $\overline{α}х\overline{β}⊥\overline{α}$, $\overline{α}х\overline{β}⊥\overline{β}$
  3. $(\overline{α}х\overline{β},\overline{α},\overline{β})$ и $(\overline{i},\overline{j},\overline{k})$ одинаково ориентированы (рис.\circ=4\cdot 3\cdot 1=12$

    Ответ: $12$.

    Вычисление векторного произведения по координатам векторов

    Из определения 1 сразу же вытекает и способ нахождения векторного произведения для двух векторов. Поскольку вектор кроме значения имеет еще и направление, находить его только при помощи скалярной величины невозможно. Но помимо него существует еще способ нахождения с помощью координат данных нам векторов.

    Пусть нам даны векторы $\overline{α}$ и $\overline{β}$, которые будут иметь координаты $(α_1,α_2,α_3)$ и $(β_1,β_2,β_3)$, соответственно. Тогда вектор векторного произведения (а именно его координаты) можно найти по следующей формуле:

    $\overline{α}х\overline{β}=\begin{vmatrix}\overline{i}&\overline{j}&\overline{k}\\α_1&α_2&α_3\\β_1&β_2&β_3\end{vmatrix}$

    Иначе, раскрывая определитель, получим следующие координаты

    $\overline{α}х\overline{β}=(α_2 β_3-α_3 β_2,α_3 β_1-α_1 β_3,α_1 β_2-α_2 β_1)$

    Пример 2

    Найти вектор векторного произведения коллинеарных векторов $\overline{α}$ и $\overline{β}$ с координатами $(0,3,3)$ и $(-1,2,6)$.

    Решение .

    Воспользуемся формулой, приведенной выше. Получим

    $\overline{α}х\overline{β}=\begin{vmatrix}\overline{i}&\overline{j}&\overline{k}\\0&3&3\\-1&2&6\end{vmatrix}=(18-6)\overline{i}-(0+3)\overline{j}+(0+3)\overline{k}=12\overline{i}-3\overline{j}+3\overline{k}=(12,-3,3)$

    Ответ: $(12,-3,3)$.

    Свойства векторного произведения векторов

    Для произвольных смешанных трех векторов $\overline{α}$, $\overline{β}$ и $\overline{γ}$, а также $r∈R$ справедливы следующие свойства:

    Пример 3

    Найдите площадь параллелограмма, вершины которого имеют координаты $(3,0,0)$, $(0,0,0)$, $(0,8,0)$ и $(3,8,0)$.

    Решение .

    Вначале изобразим данный параллелограмм в координатном пространстве (рис.5):

    Рисунок 5. Параллелограмм в координатном пространстве. Автор24 — интернет-биржа студенческих работ

    Видим, что две стороны этого параллелограмма построены с помощью коллинеарных векторов с координатами $\overline{α}=(3,0,0)$ и $\overline{β}=(0,8,0)$.2}=24$

    Vector Product — обзор

    3.4 Быстрое тензорно-векторное произведение Ханкеля

    Общие тензорно-векторные произведения высокого порядка и большого размера без структур стоят дорого. Для квадратного тензора A порядка m и размерности n вычислительная сложность тензорно-векторного произведения Axm − 1 или Axm равна Onm. Однако, поскольку тензоры Ганкеля и антициркулянтные тензоры имеют низкие степени свободы, можно ожидать, что существует гораздо более быстрый алгоритм для произведений тензора Ганкеля на вектор.Мы сосредоточимся на следующих двух произведениях тензор-вектор:

    y = A × 2×2 ⋯ × mxm, α = A × 1×1 × 2×2 ⋯ × mxm,

    , которые будут полезны для приложений.

    Быстрый алгоритм для антициркулянтных тензорно-векторных произведений легко вывести из теоремы 3.1. Пусть C = DFnm — тензор антициркуляции порядка m и размерности n со сжатым порождающим вектором c . Тогда для векторов x2, x3,…, xm∈ℂn имеем

    y = C × 2×2 ⋯ × mxm = FnD × 2Fnx2 ⋯ × mFnxm.

    Напомним, что diagD = ifftc и F n v = fft ( v ), где fft — это символ типа Matlab для быстрого преобразования Фурье.Таким образом, быстрая процедура для вектора y :

    y = fftifftc⋅ ∗ fftx2. ∗ ⋯. ∗ fftxm,

    , где u . * v умножает два вектора покомпонентно. Аналогично, для векторов x1, x2,…, xm∈ℂn имеем

    α = C × 1×1 × 2×2 ⋯ × mxm = D × 1Fnx1 × 2Fnx2 ⋯ × mFnxm,

    и быстрая процедура для скаляра α равно

    α = ifftc⊺fftx1. ∗ fftx2. ∗ ⋯. ∗ fftxm.

    Поскольку вычислительная сложность БПФ или ОБПФ является Onlogn, оба типа антициркулянтных тензорно-векторных произведений могут быть реализованы со сложностью Om + 1nlogn, что даже намного быстрее, чем произведение общего n -by- n матрица и вектор.

    Для быстрого алгоритма произведений тензора Ганкеля на вектор, мы встраиваем тензор Ганкеля в больший антициркулянтный тензор. Пусть ℋ∈ℂn1 × n2 × ⋯ × nm — тензор Ганкеля с порождающим вектором h . Обозначим Cℋ антициркулянтный тензор порядка m и размерности dℋ = n1 + n2 + ⋯ + nm − m + 1 со сжатым порождающим вектором h . Тензор H находится в верхнем левом лобовом углу Cℋ, как показано на рис. 3.1. Следовательно, мы имеем

    Рисунок 3.1. Вложение тензора Ганкеля в антициркулянтный тензор.

    Cℋ × 2 [x20] ⋯ × m [xm0] = [ℋ × 2×2 ⋯ × mxm †], Cℋ × 1 [x10] ⋯ × m [xm0] = ℋ × 1×1 ⋯ × mxm,

    где † указывает часть, не представляющая интереса. Следовательно, произведение тензора на вектор Ганкеля может быть реализовано путем умножения большего антициркулянтного тензора на некоторые увеличенные векторы. Следовательно, быстрая процедура для y = ℋ × 2×2… × mxm:

    x ~ p = xp⊺0,0,…, 0⏟dℋ − np⊺, p = 2,3,…, m, y ~ = fftiffth. ∗ fftx ~ 2. ∗ ⋯. ∗ fftx ~ m, y = y ~ 0: n1−1,

    и что для α = ℋ × 1×1 × 2×2 ⋯ × mxm равно

    x ~ p = xp⊺0,0 ,…, 0⏟dℋ − np⊺, p = 1,2,…, m, α = iffth⊺fftx ~ 1.∗ fftx ~ 2, ∗ ⋯. ∗ fftx ~ m.

    Кроме того, вычислительная сложность составляет Om + 1dℋlogdℋ. Когда тензор Ганкеля представляет собой тензор квадратов, сложность составляет Om2nlogmn, что намного меньше сложности Onm неструктурированных произведений.

    Помимо низкой вычислительной сложности, наш алгоритм для тензорно-векторных произведений Ганкеля имеет еще два преимущества. Во-первых, эта схема оптимальна в том смысле, что в ней нет лишних элементов. Не требуется явно формировать тензор Ганкеля, нужен только порождающий вектор.Еще одно преимущество состоит в том, что наш алгоритм обрабатывает тензор как ансамбль, а не умножает тензор на векторы режим за режимом.

    Для случаев BCCB и BHHB у нас также есть быстрые алгоритмы для тензорно-векторных произведений. Пусть C — тензор BCCB порядка m со сжатой порождающей матрицей C∈ℂn × N. Так как C можно диагонализовать с помощью F N F n

    C = DFN⊗Fnm,

    , мы имеем для векторов x2, x3,…, xm∈ℂnN

    y = C × 2×2 ⋯ × mxm = FN⊗FnD × 2FN⊗Fnx2 ⋯ × mFN⊗Fnxm.

    Напомним оператор векторизации и обратный к нему оператор

    vecA = A:, 0⊤A:, 1⊤… A:, N − 1⊤⊤∈ℂnN, vecn, N − 1v = v0: n − 1vn: 2n− 1… vN − 1n: Nn − 1∈ℂn × N,

    для матрицы A∈ℂn × N и вектора v∈ℂnN, и соотношение

    B⊗Av = vecA⋅vecn, N − 1v⋅B⊤.

    Следовательно, FN⊗Fnxp = vecFn⋅vecn, N − 1xp⋅FN может быть вычислено с помощью двухмерного быстрого преобразования Фурье (FFT2). Тогда быстрая процедура для y = C × 2×2… × mxm:

    Xp = vecn, N − 1xp, p = 2,3,…, m, Y = fft2ifft2C. ∗ fft2X2. ∗ ⋯. ∗ fft2Xm, y = vecY ,

    , а для α = C × 1×1 × 2×2… × mxm равно

    Xp = vecn, N − 1xp, p = 1,2,…, m, α = ifft2C, fft2X1.∗ fft2X2. ∗ ⋯. ∗ fft2Xm,

    , где 〈 A , B 〉 обозначает

    AB = ∑j, kAjkBjk.

    Для тензора BHHB H с порождающей матрицей H мы выполняем вложение дважды. Сначала мы встраиваем каждый блок Ханкеля в больший антициркулянтный блок, а затем мы встраиваем блочный тензор Ганкеля с антициркуляционными блоками в тензор BCCB Cℋ в блочном смысле. Обратите внимание, что сжатая порождающая матрица Cℋ . — это в точности порождающая матрица H. Следовательно, у нас есть быстрая процедура для y = ℋ × 2×2 ⋯ × mxm:

    X ~ p = vecnp, Np − 1xpOOO⏟N1 + N2 + ⋯ + Nm − m + 1n1 + n2 + ⋯ + нм − m + 1, p = 2,3,…, m, Y ~ = fft2ifft2H.∗ fft2X ~ 2. ∗ ⋯. ∗ fft2X ~ m, y = vecY ~ 0: n1−1,0: N1−1.

    Иногда в приложениях нет необходимости в векторизации в последней строке, поскольку и y содержат одинаковую информацию. У нас также есть быстрая процедура для α = ℋ × 1×1 × 2×2 ⋯ × mxm:

    X ~ p = vecnp, Np − 1xpOOO⏟N1 + N2 + ⋯ + Nm − m + 1n1 + n2 + ⋯ + nm − m + 1, p = 1,2,…, m, α = ifft2H, fft2X ~ 1. ∗ fft2X ~ 2. ∗ ⋯. ∗ fft2X ~ m.

    Точно так же мы можем вывести быстрые алгоритмы для высокоуровневых блочных тензорно-векторных произведений Ганкеля, используя многомерное БПФ.

    2.8: Произведения векторов (Часть 1)

    Вектор можно умножить на другой вектор, но нельзя разделить на другой вектор. Есть два вида произведений векторов, широко используемых в физике и технике. Один из видов умножения — это скалярное умножение двух векторов . В результате скалярного произведения двух векторов получается число (скаляр), как указывает его название. Скалярные произведения используются для определения отношений между работой и энергией. Например, работа, которую сила (вектор) выполняет с объектом, вызывая его смещение (вектор), определяется как скалярное произведение вектора силы на вектор смещения.Совсем другой вид умножения — это векторное умножение векторов . Получение векторного произведения двух векторов возвращает в результате вектор, как следует из его названия. Векторные произведения используются для определения других производных векторных величин. Например, при описании вращений векторная величина, называемая крутящим моментом , определяется как векторное произведение приложенной силы (вектора) и ее плеча рычага (вектора). Важно различать эти два вида векторных умножений, потому что скалярное произведение — это скалярная величина, а векторное произведение — это векторная величина.

    Скалярное произведение двух векторов (скалярное произведение)

    Скалярное умножение двух векторов дает скалярное произведение.

    Определение: скалярное произведение (скалярное произведение)

    Скалярное произведение \ (\ vec {A} \; \ cdotp \ vec {B} \) двух векторов \ (\ vec {A} \) и \ (\ vec {B} \) — это число, определяемое уравнение

    \ [\ vec {A} \; \ cdotp \ vec {B} = AB \ cos \ varphi, \ label {2.27} \]

    , где \ (\ phi \) — угол между векторами (показано на рисунке \ (\ PageIndex {1} \)).Скалярное произведение также называется скалярным произведением из-за точечной записи, которая его указывает.

    В определении скалярного произведения направление угла \ (\ varphi \) не имеет значения, а \ (\ varphi \) можно измерить от одного из двух векторов к другому, потому что \ (\ cos \ varphi \ ) = \ (\ cos (- \ varphi) \) = \ (cos (2 \ pi — \ varphi) \). Скалярное произведение — отрицательное число, когда 90 ° <\ (\ varphi \) ≤ 180 °, и положительное число, когда 0 ° ≤ \ (\ phi \) <90 °. Более того, скалярное произведение двух параллельных векторов равно \ (\ vec {A} \ cdotp \ vec {B} \) = AB cos 0 ° = AB, а скалярное произведение двух антипараллельных векторов равно \ (\ vec {A} \; \ cdotp \ vec {B} \) = AB cos 180 ° = −AB.{2} \ label {2.28} \]

    Рисунок \ (\ PageIndex {1} \): скалярное произведение двух векторов. (а) Угол между двумя векторами. (b) Ортогональная проекция A вектора \ (\ vec {A} \) на направление вектора \ (\ vec {B} \). (c) Ортогональная проекция B вектора \ (\ vec {B} \) на направление вектора \ (\ vec {A} \).

    Пример \ (\ PageIndex {1} \): скалярное произведение

    Для векторов, показанных на рисунке 2.3.6, найдите скалярное произведение \ (\ vec {A} \; \ cdotp \ vec {F} \).

    Стратегия

    Из рисунка 2.3.6, модули векторов \ (\ vec {A} \) и \ (\ vec {B} \) равны A = 10,0 и F = 20,0. {o} = 51.{2} = \ hat {k} \; \ cdotp \; \ hat {k} = 1 \ ldotp \ label {2.30} \]

    Скалярное произведение \ (\ vec {A} \; \ cdotp \ vec {B} \) также можно интерпретировать как произведение B на проекцию A \ (_ {\ parallel} \) вектора \ (\ vec {A} \) на направление вектора \ (\ vec {B} \) (рисунок \ (\ PageIndex {1} \) (b)) или произведение A на проекцию B \ (_ {\ parallel } \) вектора \ (\ vec {B} \) на направление вектора \ (\ vec {A} \) (Рисунок \ (\ PageIndex {1} \) (c)):

    \ [\ begin {split} \ vec {A} \; \ cdotp \ vec {B} & = AB \ cos \ varphi \\ & = B (A \ cos \ varphi) = BA _ {\ parallel} \\ & = A (B \ cos \ varphi) = AB _ {\ parallel} \ ldotp \ end {split} \]

    Например, в прямоугольной системе координат на плоскости скалярная x-компонента вектора — это его скалярное произведение с единичным вектором \ (\ hat {i} \), а скалярная y-компонента вектора — это его скалярная составляющая. скалярное произведение с единичным вектором \ (\ hat {j} \):

    \ [\ begin {cases} \ vec {A} \; \ cdotp \; \ hat {i} = | \ vec {A} || \ hat {i} | \ cos \ theta_ {A} = A \ cos \ theta_ {A} = A \ cos \ theta_ {A} = A_ {x} \\ \ vec {A} \; \ cdotp \; \ hat {j} = | \ vec {A} || \ hat {j} | \ cos (90 ^ {o} — \ theta_ {A}) = A \ sin \ theta_ {A} = A_ {y} \ end {cases} \]

    Скалярное умножение векторов коммутативное,

    \ [\ vec {A} \; \ cdotp \ vec {B} = \ vec {B} \; \ cdotp \ vec {A}, \ label {2.31} \]

    и подчиняется закону о распределении доходов:

    \ [\ vec {A} \; \ cdotp (\ vec {B} + \ vec {C}) = \ vec {A} \; \ cdotp \ vec {B} + \ vec {A} \; \ cdotp \ vec {C} \ ldotp \ label {2.32} \]

    Мы можем использовать законы коммутативности и распределения для вывода различных соотношений для векторов, таких как выражение скалярного произведения двух векторов через их скалярные компоненты.

    Упражнение 2.12

    Для вектора \ (\ vec {A} = A_ {x} \; \ hat {i} + A_ {y} \; \ hat {j} + A_ {z} \; \ hat {k} \) в прямоугольная система координат, используйте уравнение \ ref {2.29} через уравнение \ ref {2.32}, чтобы показать, что \ (\ vec {A} \; \ cdotp \ hat {i} = A_ {x} \ vec {A} \; \ cdotp \; \ hat {j} = A_ {y} \) и \ (\ vec {A} \; \ cdotp \; \ hat {k} = A_ {z} \).

    Если векторы в уравнении \ ref {2.27} заданы в форме их векторных компонентов,

    \ [\ vec {A} = A_ {x} \; \ hat {i} + A_ {y} \; \ hat {j} + A_ {z} \; \ hat {k} \; и \ vec {B} = B_ {x} \; \ hat {i} + B_ {y} \; \ hat {j} + B_ {z} \; \ hat {k}, \]

    , мы можем вычислить их скалярное произведение следующим образом:

    \ [\ begin {split} \ vec {A} \; \ cdotp \ vec {B} & = (A_ {x} \; \ hat {i} + A_ {y} \; \ hat {j} + A_ {z} \; \ hat {k}) \; \ cdotp (B_ {x} \; \ hat {i} + B_ {y} \; \ hat {j} + B_ {z} \; \ hat {k}) \\ & = A_ {x} B_ {x } \; \ hat {i} \; \ cdotp \; \ hat {i} + A_ {x} B_ {y} \; \ hat {i} \; \ cdotp \; \ hat {j} + A_ {x} B_ {z} \; \ hat {i} \; \ cdotp \; \ hat {k} \\ & + A_ {y} B_ {x} \; \ hat {j} \ cdotp \; \ hat {i} + A_ {y} B_ {y} \; \ hat {j} \; \ cdotp \; \ hat {j} + A_ {y} B_ {z} \; \ hat {j} \ cdotp \; \ hat {k} \\ & + A_ {z} B_ {x} \; \ hat {k} \; \ cdotp \; \ hat {i} + A_ {z} B_ {y} \; \ hat {k} \; \ cdotp \; \ hat {j} + A_ {z} B_ {z} \; \ hat {k} \; \ cdotp \; \ hat {k} \ ldotp \ end {split} \]

    Поскольку скалярные произведения двух разных единичных векторов осей дают ноль, а скалярные произведения единичных векторов сами с собой дают единицу (см. Уравнение \ ref {2.29} и Equation \ ref {2.30}) в этом выражении есть только три ненулевых члена. Таким образом, скалярное произведение упрощается до

    \ [\ vec {A} \; \ cdotp \ vec {B} = A_ {x} B_ {x} + A_ {y} B_ {y} + A_ {z} B_ {z} \ ldotp \ label {2.33} \]

    Мы можем использовать уравнение \ ref {2.33} для скалярного произведения в терминах скалярных компонентов векторов, чтобы найти угол между двумя векторами. Когда мы разделим уравнение \ ref {2.27} на AB, мы получим уравнение для cos \ (\ varphi \), в которое мы подставим уравнение \ ref {2.33}:

    \ [\ cos \ varphi = \ frac {\ vec {A} \; \ cdotp \ vec {B}} {AB} = \ frac {A_ {x} B_ {x} + A_ {y} B_ {y} + A_ {z} B_ {z}} {AB} \ ldotp \ label { 2.34} \]

    Угол \ (\ varphi \) между векторами \ (\ vec {A} \) и \ (\ vec {B} \) получается путем взятия обратного косинуса выражения в уравнении \ ref {2.34}.

    Пример \ (\ PageIndex {2} \)

    Три собаки тянут палку в разные стороны, как показано на рисунке \ (\ PageIndex {2} \). Первая собака тянет с силой \ (\ vec {F} _ {1} \) = (10.0 \ (\ hat {i} \) — 20.4 \ (\ hat {j} \) + 2.0 \ (\ hat {k} \)) N, вторая собака тянет с силой \ (\ vec {F} _ { 2} \) = (−15.0 \ (\ hat {i} \) — 6.2 \ (\ hat {k} \)) N, а третья собака тянет с силой \ (\ vec {F} _ {3} \ ) = (5,0 \ (\ hat {i} \) + 12,5 \ (\ hat {j} \)) N. Каков угол между силами \ (\ vec {F} _ {1} \) и \ (\ vec {F} _ {2} \)?

    Рисунок \ (\ PageIndex {2} \): Три собаки играют палкой.

    Стратегия

    Компоненты вектора силы \ (\ vec {F} _ {1} \) равны F 1x = 10,0 Н, F 1y = −20.{o} \ ldotp \]

    Значение

    Обратите внимание, что когда векторы задаются в терминах единичных векторов осей, мы можем найти угол между ними, не зная специфики географических направлений, которые представляют единичные векторы. Здесь, например, направление + x может быть на восток, а направление + y — на север. Но угол между силами в задаче будет таким же, если + x-направление направлено на запад, а + y-направление — на юг.

    Упражнение 2.13

    Найдите угол между силами \ (\ vec {F} _ {1} \) и \ (\ vec {F} _ {3} \) в примере \ (\ PageIndex {2} \).

    Пример \ (\ PageIndex {3} \): Работа силы

    Когда сила \ (\ vec {F} \) тянет объект и вызывает его смещение \ (\ vec {D} \), мы говорим, что сила выполняет работу. Количество работы, совершаемой силой, — это скалярное произведение \ (\ vec {F} \; \ cdotp \ vec {D} \). Если палка в примере \ (\ PageIndex {2} \) на мгновение сдвинется и сместится на вектор \ (\ vec {D} \) = (−7.9 \ (\ hat {j} \) — 4,2 \ (\ hat {k} \)) см, сколько работы выполняет третья собака в Примере \ (\ PageIndex {2} \)?

    Стратегия

    Мы вычисляем скалярное произведение вектора смещения \ (\ vec {D} \) с вектором силы \ (\ vec {F} _ {3} \) = (5.0 \ (\ hat {i} \) + 12.5 \ ( \ hat {j} \)) N, тяга третьей собаки. Используем W 3 для обозначения работы, совершаемой силой \ (\ vec {F} _ {3} \) при перемещении \ (\ vec {D} \).

    Решение

    Расчет работы — это прямое приложение скалярного произведения:

    \ [\ begin {split} W_ {3} & = \ vec {F} _ {3} \; \ cdotp \ vec {D} = F_ {3x} D_ {x} + F_ {3y} D_ {y} + F_ {3z} D_ {z} \\ & = (5.0 \; N) (0,0 \; см) + (12,5 \; N) (- 7,9 \; см) + (0,0 \; N) (- 4,2 \; см) \\ & = -98,7 \; N \; \ cdotp см \ ldotp \ end {split} \]

    Значение

    Единица работы в системе СИ называется джоуль (Дж), где 1 Дж = 1 Н · м. Единицу см · Н можно записать как 10 −2 м · N = 10 −2 Дж, поэтому ответ можно выразить как W 3 = −0,9875 Дж ≈ −1,0 Дж.

    Упражнение 2.14

    Сколько работы выполняется первой собакой и второй собакой в ​​Примере \ (\ PageIndex {2} \) над смещением в Примере \ (\ PageIndex {3} \)?

    Векторное произведение — статья энциклопедии

    Векторное произведение , также известное как перекрестное произведение , является антисимметричным произведением A × B = — B × A двух векторов A и B в трехмерном евклидовом пространстве Космос .Векторное произведение снова является трехмерным вектором. Векторное произведение широко используется во многих областях математики, механики, электромагнетизма, гравитационных полей и т. Д.

    Определение

    Для двух векторов, A и B in, векторное произведение представляет собой вектор длиной AB sin θ AB , где A — длина A , B — длина B , а θ AB — меньший (невозвратный) угол между A и B .Направление векторного произведения перпендикулярно (или перпендикулярно) плоскости, содержащей векторы A и B , и следует правилу правой руки,

    , где a N — единичный вектор, нормальный к плоскости, охватываемой A и B в направлении правой линейки.

    Напомним, что длина вектора — это квадратный корень из скалярного произведения вектора на себя, A ≡ | A | = ( A A ) 1/2 и аналогично для длины B .Единичный вектор по определению имеет длину один.

    Из антисимметрии A × B = — B × A следует, что перекрестное (векторное) произведение любого вектора на себя (или другой параллельный или антипараллельный вектор) равно нулю, потому что A × A = — A × A и единственная величина, равная самому минусу, — это ноль. В качестве альтернативы, это можно вывести из того факта, что sin (0) = 0 (параллельные векторы) и sin (180) = 0 (антипараллельные векторы).

    Правое правило

    (PD) Изображение: D.E. Volk
    Рис. 1. Схема, показывающая направление A × B . Плоскость, содержащая A и B , является плоскостью x y , A × B расположена вдоль оси z .

    Диаграмма на рис. 1 иллюстрирует направление A × B , которое следует правилу правой руки. Если направить пальцы правой руки в сторону головы вектора A (запястье в начале координат), а затем согнуть их в направлении B , вытянутый большой палец будет указывать в направлении A × В .

    Другая формулировка перекрестного произведения

    Вместо угла и перпендикулярного единичного вектора часто используется другая форма векторного произведения. В альтернативном определении векторы должны быть выражены относительно декартовой (ортонормированной) системы координат a x , a y и a z of.

    В отношении этого кадра мы пишем A = ( A x , A y , A z ) и B = ( B x , B y , B z ).потом

    A × B = ( A y B z A z B y ) a x + ( A z B x A x B z ) a y + ( A x B y A y B x ) a z .

    Эту формулу можно записать более кратко, если ввести определитель:

    где обозначает определитель матрицы. Этот определитель должен оцениваться по первой строке, иначе уравнение не имеет смысла.

    Геометрическое изображение длины

    Повторяем, что длина векторного произведения векторов A и B равна

    , потому что a N по определению имеет длину 1.

    CC Image
    Рис. 2. Длина скрещенного произведения A × B равна площади параллелограмма со сторонами a и b , сумме площадей пунктирного и штрихового треугольника.

    Используя правило геометрии средней школы: площадь S треугольника равна его основанию в a , умноженному на его половину высоты d , мы видим на рис. 2, что площадь S пунктирного треугольника равна :

    , поскольку, как следует из рис.2:

    Следовательно | A × B | = 2 S . Поскольку пунктирный треугольник со сторонами a , b и c конгруэнтен пунктирному треугольнику, площадь пунктирного треугольника равна площади пунктирного треугольника и длине 2 S креста. произведение равно сумме площадей пунктирного и штрихового треугольников. В заключение: площадь параллелограмма, образованного векторами A, и B , равна длине A, × B .

    Применение: объем параллелепипеда

    Объем V параллелепипеда, показанного на рис. 3, определяется выражением

    Действительно, помните, что объем параллелепипеда равен площади S его основания, умноженной на его высоту h , V = Sh . Выше было показано, что если

    Высота h параллелепипеда — это длина проекции вектора A на D .Скалярное произведение между A и D равно | D | умноженное на длину h ,

    , так что V = ( B × C ) ⋅ A = A ⋅ ( B × C ).

    Интересно отметить, что V может быть задано определителем, который содержит компоненты A , B и C относительно декартовой системы координат,

    Из свойств перестановки определителя следует

    Произведение A ⋅ ( B × C ) часто называют тройным скалярным произведением .Это псевдоскаляр . Термин скаляр относится к тому факту, что тройное произведение инвариантно при одном и том же одновременном вращении на A, , B и C . Термин псевдо относится к тому факту, что одновременная инверсия A → — A , B → — B и C → — C преобразует тройное произведение в минус.

    Произведение в виде линейной карты

    Учитывая фиксированный вектор n , применение n × является линейным,

    Это означает, что n × r может быть записано как произведение матрица-вектор,

    Матрица N имеет общий элемент

    где ε αβγ — антисимметричный символ Леви-Чивиты.Следует, что

    Связь с бесконечно малым вращением

    Вращение вектора r вокруг единичного вектора на угол φ отправляет r на r ′. Повернутый вектор связан с исходным вектором соотношением

    Предположим теперь, что φ бесконечно мала и равна Δφ, т.е. квадраты и более высокие степени Δφ пренебрежимо малы по отношению к Δφ, тогда

    так, чтобы

    Линейный оператор отображения; он известен как генератор бесконечно малого вращения вокруг .

    Обобщение

    С несколько более абстрактной точки зрения можно определить векторное произведение как элемент антисимметричного подпространства 9-мерного пространства тензорного произведения. Это антисимметричное подпространство размерности 3.

    В общем случае антисимметричное подпространство тензорной степени k кратно размерно. Элементы такого пространства часто называют клином или экстерьерными изделиями, написанными как

    Антисимметричное подпространство двумерного тензорного пространства произведения имеет размерность

    .

    Последнее число равно 3, только если n = 3. Например, для n = 2 или 4 антисимметричные подпространства имеют размерность 1 и 6 соответственно.

    Следовательно, если рассматривать векторное произведение с точки зрения антисимметричных подпространств, то это «совпадение», в котором произведение снова находится.

    В перекрестном произведении отсутствует свойство правильного вектора, заключающееся в том, что он меняет знак при инверсии [оба множителя перекрестного произведения меняют знак и (−1) × (−1) = 1].Вектор, не меняющий знака при инверсии, называется аксиальным вектором или псевдовектором. Следовательно, кросс-произведение — это псевдовектор. Вектор, который действительно меняет знак, в этом контексте часто называют полярным вектором.

    Внешняя ссылка

    Самый первый учебник по векторному анализу ( Векторный анализ: учебник для студентов, изучающих математику и физику, основанный на лекциях Дж. Уилларда Гиббса Эдвина Бидвелла Уилсона, издательство Йельского университета, Нью-Хейвен, 1901 г. ) можно найти здесь.В этой книге вводятся синонимы: косое произведение, кросс-произведение и векторное произведение.

    Векторное произведение

    Векторное произведение
    следующий: Центр масс Вверх: Вращательное движение Пред .: Является ли вращение вектором?


    Векторное произведение Мы видели ранее в разд. 3.10, что можно мультипликативно объединить два вектора, с помощью скалярного произведения , чтобы образуют скаляр. Напомним, что скалярное произведение двух векторов а также определено
    (319)

    где — угол между направлениями и.

    Можно ли также мультипликативно объединить два вектора для образования третьего (некомпланарного) вектора? Оказывается, что этой цели можно достичь с помощью так называемого векторное произведение . По определению векторное произведение , двух векторов а также имеет величина

    (320)

    Направление взаимно перпендикулярно к и , в смысле, заданном правилом правого захвата, когда вектор поворачивается на вектор (направление вращения такой, чтобы угол поворота был меньше).См. Рис. 70. В координатной форме
    (321)

    Рисунок 70: Векторное произведение.

    Есть ряд довольно очевидных следствий приведенного выше определения. Во-первых, если вектор параллельно вектору, так что мы можем написать , тогда векторное произведение имеет нулевую величину . Самый простой способ увидеть следует отметить, что если и параллельны, то угол между они равны нулю, следовательно, величина векторного произведения, , должен также равняться нулю (так как ).Во-вторых, имеет значение порядок умножения. Таким образом, , а не эквивалентно . По факту, как видно из уравнения. (321),

    (322)

    Другими словами, имеет ту же величину, что и , но указывает на диаграмму противоположное направление.

    Теперь, когда мы определили векторное произведение двух векторов, давайте найдем применение этой концепции. На рисунке 71 показано твердое тело, вращающееся с угловой скоростью. .Для ради простоты ось вращения, которая проходит параллельно , является предполагается, что он проходит через начало нашей системы координат. Точка, положение которой вектор, представляет собой общую точку внутри тела. Какая скорость вращения в точке? Ну, величина этой скорости просто

    (323)

    где — перпендикулярное расстояние точки от оси вращения, а угол между направлениями а также .Направление скорости — на страницу. Другой способ сказать это: направление скорости взаимно перпендикулярно направлениям а также , в смысле, указанном правилом правого захвата, когда повернут на (на угол меньше чем). Следует, что мы можем написать
    (324)

    Отметим, кстати, что направление вектора угловой скорости указывает ориентацию оси вращения — однако в этом направлении фактически ничего не движется; на самом деле все движение перпендикулярно направлению .
    Рисунок 71: Жесткое вращение.


    следующий: Центр масс Вверх: Вращательное движение Пред .: Является ли вращение вектором?
    Ричард Фицпатрик 2006-02-02

    Вычислить векторное произведение — определение, свойства и примеры

    Векторное произведение

    Вектор имеет как направление, указанное стрелкой, так и величину, указанную длиной.Векторное произведение — это комбинация двух векторов, то есть скаляра и вектора. Следовательно, у нас есть два способа умножения векторов. Во-первых, это скалярное произведение векторов, также известное как скалярное произведение. Другой — это векторное произведение векторов, также известное как векторное произведение. К концу этого мы, несомненно, сможем определить и вычислить векторное произведение, когда два вектора будут представлены в декартовой форме, и изучить географические применения этого.

    Определение векторного произведения

    Теперь вы можете определить векторное произведение? Вы можете начать с примера.Вот два вектора (a и b), а угол между ними представлен как.

    (Изображение будет добавлено в ближайшее время)

    К настоящему времени мы знаем, что когда два вектора умножаются, результатом всегда является вектор. Итак, чтобы получить вектор, нам сначала нужно указать направление. И по определению направление векторного произведения находится под прямым углом как к a, так и к b. Это также означает, что они расположены под прямым углом даже к плоскости, в которой лежат a и b.

    (Изображение будет добавлено в ближайшее время)

    Таким образом, у нас есть два варианта. Чтобы сделать этот выбор, мы можем воспользоваться правилом правого винта. Согласно этому правилу направление векторного произведения должно совпадать с направлением поворота отвертки, то есть от a к b.

    (Изображение будет добавлено в ближайшее время)

    Векторное произведение a и b должно быть определено как: a x b = | a || b | sin \ [\ theta \] \ [\ widehat {n} \]

    Где, | a | модуль или величина a,

    | b | — модуль b

    \ [\ theta \] — угол между a и b

    \ [\ widehat {n} \] — единица векторов, перпендикулярная как a, так и b.

    Примечание. Векторное произведение также называется кросс-векторным произведением, поскольку символ векторного произведения — x

    Свойства векторного произведения

    Прежде чем продолжить, мы должны знать несколько свойств векторных произведений. Это:

    1. Порядок, в котором мы выполняем вычисления, имеет значение, так как a x b и b x a противоположны друг другу. Следовательно, векторное произведение не коммутативно.

    2. Векторное произведение всегда является распределительным по сравнению с сложением, например:

    a x (b + c) = a x b + a x c

    Это основные свойства векторного произведения, которые будут вам полезны.

    Перекрестное векторное произведение двух параллельных векторов

    Считайте два вектора (a и b) параллельными, но определение вектора не применяется к параллельным прямым, поскольку два параллельных вектора не определяют плоскость. Следовательно, векторное произведение двух параллельных векторов будет равно нулю.

    Векторное произведение двух параллельных векторов в декартовой форме

    Мы можем найти векторное произведение двух векторов в декартовой форме, например a = 3i — 2j + 7k и b = -5i + 4j — 3k, где i, j , а k — единичные векторы в направлениях осей x, y и z соответственно.Мы можем использовать формулу, которую в конце концов разработаем. Итак, сначала давайте начнем с нескольких примеров перекрестных произведений:

    Пример 1) Предположим, что мы хотим найти i x j. Теперь, поскольку они лежат вдоль осей x и y, мы можем сказать, что эти векторы перпендикулярны.

    (Изображение будет добавлено в ближайшее время)

    Здесь мы видим, что k — это единичный вектор, перпендикулярный i и j, а угол между i и j равен 90 градусов, а sin 90 градусов равен 1. С помощью ручного винта правило, мы можем найти ix j.Следовательно, i x j = | i || j | sin 900 k

    = (1) (1) (1) k

    = k

    Пример 2) Теперь, если мы найдем j x i, используя правило ручного винта, вектор, перпендикулярный j и i, будет равен -k. Следовательно, jxi = -k

    Пример 3) Нахождение ixi приведет к нулю, поскольку они перпендикулярны, а угол между ними равен 00. Следовательно, ixi = 0

    На основе этого примера перекрестного произведения мы можем резюмировать следующее:

    1. ixi = 0

    2. jxj = 0

    3. K xk = 0

    4. ixj = k

    5. jxk = i

    6. jxk = i

    7. jxj =

    8. k

    9. kxj = -i

    10. ixk = -j

    Следующий пример перекрестного произведения можно использовать для формирования формулы для поиска векторного произведения двух векторов в декартовой форме.

    a = a1i + a2j + a3k и b = b1i + b2j + b3k, тогда

    axb = (a1i + a2j + a3k) x (b1i + b2j + b3k)

    = a1i x (b1i + b2j + b3k) + a2j x (b1i + b2j + b3k) + a3k x (b1i + b2j + b3k)

    = a1i x b1i + a1i x b2j + a1i x b3k + a2j x b1i + a2j x b2j + a2j x b3k + a3k x b1 + a3k x b2j + a3k x b3k

    = a1b1i xi + a1b2i xj + a1b3i xk + a2b1j xi + a2b2j xj + a2b3j xk + a3b1k xi + a3b2k xj + a3b3k22k. из этих терминов равны нулю.Следовательно,

    a x b = (a2b3 — a3b2) i + (a3b1 — a1b3) j + (a1b2 — a2b1) k является формулой векторного произведения двух векторов, которую мы можем использовать для вычисления векторного произведения в декартовых компонентах двух векторов.

    EngArc — L — Векторное произведение (перекрестное произведение)

    EngArc — L — Векторное произведение (перекрестное произведение)


    Быстрый

    V = P × Q
    (читается как v равно векторному произведению (или перекрестному произведению) p и q)
    , где V , P и Q — векторы, а величина V равна:
    В = PQ sin θ

    Детали

    Векторное произведение, также известное как перекрестное произведение (из-за обозначения), представляет собой вектор (произвольно названный вектор C), который получается в результате определенных операций, которые принимают во внимание два других вектора (произвольно названные вектор A и вектор B).

    Общее обозначение векторного произведения двух векторов A и B записывается следующим образом:

    C = A × B

    Знак × выглядит как знак умножения, но это знак векторного произведения.

    Порядок записи A и B важен, поскольку A и B не являются коммутативными, кроме того:

    A × B = — ( B × A )

    Условия, которые должны быть выполнены

    1. Векторное произведение (вектор C) должно быть перпендикулярно плоскости, которая содержит точки A и B .
    2. Величина C должна равняться произведению величин A и B и синуса угла θ (должен быть 180 ° или меньше), образованного A и B :

      (уравнение 1) C = AB sin θ

    3. Направление C находится с помощью правила правой руки.Правило правой руки используется в основном в соответствии с векторным произведением.
    Вектор (вектор C), который удовлетворяет этим трем условиям (которые однозначно определяют его), называется векторным произведением A и B ; он представлен математическим выражением C = A × B .

    Правило правой руки

    Сформированный кулак правой рукой.Используя движение, которое можно было бы использовать для сжатия объекта, пальцы скручиваются в том же направлении, что и вращение на θ , которое приводит вектор A в соответствие с вектором B .

    Примечание. Это возможно только в том случае, если берется векторное произведение A × B . Если взять векторное произведение B × A , то пальцы скручены в том же направлении, что и вращение на θ , которое приводит вектор B в соответствие с вектором A , и вектор будет указывать в противоположном направлении.Большой палец указывает в направлении вектора C . Обратите внимание, что если A и B не происходят из одной и той же точки, их сначала следует перерисовать из одной и той же точки.

    Для C = A × B , три вектора, A , B и C — взяты в таком порядке — форма правосторонняя триада.

    Дополнительные пояснения

    Когда два вектора A и B имеют одинаковое или противоположные направления, их векторное произведение равно нулю (sin (0 °) = sin (180 °) = 0). В общем случае, когда угол θ , образованный двумя векторами, не равен ни 0 °, ни 180 °, уравнению 1 можно дать простую геометрическую интерпретацию: величина C векторного произведения A и B равна площади параллелограмма, у которого стороны A и B .Eq1 фактически равно уравнению для площади параллелограмма. Векторное произведение A × B , следовательно, останется неизменным, если B заменить вектором D , который копланарен с A и B и так, чтобы линия, соединяющая концы B и D , была параллельна A .

    Соответствующее уравнение:

    C = A × B = A × D

    Из рисунка выше видно, что площадь 1,3,5,6 равна площади 2,4,5,6.

    Векторное произведение и свойство распределения

    Распределительное свойство действительно с векторным произведением, представленным в следующем уравнении:

    A × ( B 1 + B 2 ) = A × B 1 + A × 2 34 B

    Поскольку вся структура как векторной алгебры, так и статики зависит от приведенного выше соотношения, следующий вывод:

    Без ограничения общности можно предположить, что P направлена ​​вдоль оси y fig3.9. Обозначив через Q сумму Q1 и Q2, мы опускаем перпендикуляры с концов Q, Q1 и Q2 на плоскость zx, определяя таким образом векторы Q ‘, Q’1 и Q’2. Эти векторы будем называть соответственно проекциями Q, Q1 и Q2 на плоскость zx. Вспоминая свойство, выраженное уравнением 3.3, мы отмечаем, что левый член уравнения 3.5 может быть заменен на PXQ ‘и что аналогично векторные произведения PX Q1 и PX Q2 могут быть заменены соответственно на PX Q’1 и PX Q’2. Таким образом, доказываемое соотношение можно записать в виде

    P X Q ‘= P X Q’1 + P X Q’2

    Теперь мы видим, что P X Q ‘можно получить из Q’, умножив этот вектор на скаляр P и повернув его против часовой стрелки на 90 ° в плоскости zx fig3.9b; два других векторных произведения в 3.5 ‘могут быть получены таким же образом из Q’1 и Q’2 соответственно. Теперь, поскольку проекция параллелограмма на произвольную плоскость есть параллелограмм


    Связанные
    ▪ L — Векторы
    ▪ P — Векторное произведение
    ▪ L — Момент

    Векторное произведение — Доктора математики

    В прошлый раз мы рассмотрели скалярное или точечное произведение векторов, сосредоточившись на доказательстве эквивалентности двух способов его определения.На этот раз мы посмотрим на вектор или перекрестное произведение таким же образом. Различие между скалярным произведением и перекрестным произведением отражает используемый символ, u · v против u × v , который является творческим использованием двух разных символов, которые эволюционировали для умножения чисел. Однако реальное различие заключается в том, что произведение является скаляром (числом) и вектором.

    Что такое перекрестное произведение?

    Как мы видели в прошлый раз, скалярное произведение трудно объяснить простыми геометрическими терминами, поэтому маловероятно, что кто-то изобрел бы его в такой форме.Скорее, это простой способ объединить два вектора алгебраически, который имеет свойства, подходящие для умножения, и который, как правило, включает угол между двумя векторами, в частности его косинус, что делает его очень полезным. То же самое верно и для векторного произведения, которое сложно описать геометрически, но оно (в некотором смысле) очень аккуратно алгебраически и, оказывается, включает синус угла между ними.

    Начну, как и в прошлый раз, с вопроса о сущности операции; на этот раз я должен использовать вопрос от 2016 года, который не был опубликован в Спросите доктора.Архив Math . Он начинается с геометрического описания с упоминанием алгебраической формы с использованием определителей, о которых мы поговорим ниже.

      Что на самом деле представляет собой произведение двух векторов? 
    
    Бывший. a X b = | a || b | sin (угол между векторами). u, где u - единичный вектор в направлении a X b
    
    Мой профессор физики однажды заявил, что существует  физическое определение  векторного произведения векторов. Хотя он не уточнил, я все еще думаю, что это может быть.Я попытался разбить векторы и использовать метод определителя   для оценки результата. Я внимательно изучил определитель и определение, чтобы понять. Обнаружив, что определитель представляет собой площадь параллелограмма, я пришел к выводу, что он может представлять векторную область  параллелограмма . Однако я до сих пор не понимаю, в чем заключается физический смысл. 

    Вот пример, показывающий перекрестное произведение:

    Векторы u и v образуют параллелограмм с диагональю u + v ; перекрестное произведение u × v перпендикулярно этой плоскости, и его величина равна \ (| \ mathbf {u} | | \ mathbf {v} | \ sin (\ theta) \), что является площадью этот параллелограмм.

    Я ответил, сначала сославшись на три страницы, которые мы рассмотрим ниже и в следующий раз, а затем продолжил:

     Они последовательно дают более подробные ответы на вопрос о перекрестном произведении. Я думаю, что ваше описание как «векторная площадь параллелограмма» (или « направленная область ») подходит. Я не знаю, что ваш профессор имел в виду под «физическим определением»; он * определен * математически, а не физикой, но может * применяться * в физике различными способами, которые оправдывают его использование.Он встречается в нескольких довольно разных областях физики, поэтому ни одну из них нельзя назвать значением перекрестного произведения в целом.
    
    Может быть, тебе стоит спросить своего профессора! 

    Термин Авинаша «векторная площадь», который я предложил назвать «направленной областью», относится к тому факту, что величина величины перекрестного произведения представляет собой площадь параллелограмма, определяемого векторами, а его направление перпендикулярно к плоскость двух векторов.

    В качестве особенно простого примера рассмотрим векторное произведение единичных векторов i и j , которое представляет собой вектор, перпендикулярный обоим, с величиной 1, площадью квадрата 1 на 1:

    Авинаш ответил,

     Отвечая на мой вопрос, вы упомянули "направленную зону".Я немного сбит с толку, что касается , как площадь, являющуюся скаляром, можно назвать вектором .
    
    Я знаю, я говорил о векторной области, и вы можете предположить, что я знаю об этом, но правда в том, что мне пришлось принять этот факт как есть.
    
    Я надеюсь, что вы дадите мне убедительное объяснение векторной площади. 

    Я ответил:

     Что ж, поэтому мы не называем это просто областью!  Сама площадь является скаляром ; но мы можем связать это с направлением, и бывает, что перекрестный продукт естественным образом объединяет их.3)
      и обозначается символом x.
    
    Эта направленная площадь равна площади , умноженной на единичный вектор, перпендикулярный плоскости  (которая определяет направление плоскости). 

    Фактически используется термин «векторная область», хотя и не обычно в этом контексте.

     Мы также можем говорить о « подписанной области », которая тесно связана; это обычно возникает в связи с формулой для площади области, заключенной в многоугольник, которая дает положительную площадь, если вы двигаетесь против часовой стрелки, но отрицательную площадь в другом направлении.Эта формула тесно связана с перекрестным произведением. Вот два обсуждения этого:
    
      Четырехугольник
      http://mathforum.org/library/drmath/view/60583.html
    
      Формула геометрического доказательства площади треугольника
      http://mathforum.org/library/drmath/view/72141.html 

    Эта последняя ссылка упоминает перекрестный продукт и дает вводное доказательство того, что мы увидим позже; Я ранее обсуждал это в многоугольных координатах и ​​областях.

    Авинаш преследовал вопрос:

     Спасибо за ответ.Вы мне очень помогли, но не могли бы вы объяснить , почему мы связываем направление с областью?  Трудно придумать идею сочетания площади с направлением.
    Я имею в виду, , как кто-то мог подумать об использовании направлений в области , скалярной величины? Это из-за приложений в поиске флюса или чего-то подобного? 

    Это суть вопроса: Зачем все это вообще?

    Я сказал,

     Первая причина, вероятно, в том, что когда мы, , ищем способ сделать вектор, перпендикулярный плоскости  двух заданных векторов, просто так получилось, что величина перекрестного произведения, которое выполняет эту работу, равна площади.Так что мы принимаем это как подарок! Дело не в том, что мы стремились к этой цели; нам его просто вручили как хороший результат.
    
    Также оказывается, что эта идея полезна, как вы упомянули, для таких понятий, как поток. Кто-то мог подумать об этом первым; Я не знаю. Но как бы то ни было, это хорошая идея. Все работает очень аккуратно. 

    По мере того, как мы рассмотрим нижеприведенное доказательство, мы увидим, что то, что я сказал, совершенно верно. Авинаш заключил:

     Спасибо, спасибо, спасибо, огромное спасибо за то, что ответили на все мои вопросы без ропота.Если бы я задал все эти вопросы своему профессору математики, он счел бы это «глупым» и «неуместным». Рад, что члены Dr.Math всегда готовы помочь. 

    На что я ответил:

     Пожалуйста.
    
    Мне действительно нравятся странные вопросы, на которые действительно нет ответов, но которые заставляют вас вникать в вещи глубже! Я не получаю их на уроках в достаточном количестве. 

    Теперь давайте посмотрим на первый из заархивированных ответов, на которые я направил Авинаша.

    Как (и зачем) изобрести два продукта

    Этот вопрос от 2012 г .:

     Скрещенные скалярами и векторами
    
    Я никогда по-настоящему не понимал, почему скалярные произведения и перекрестные произведения были определены именно так. Почему результат скалярного произведения - скаляр, а результат векторного произведения - вектор? 
    
    Я просмотрел множество книг и поискал в сети, но, кажется, никто не дает этому правдоподобного объяснения. 

    Доктор Джерри ответил, начиная с скалярного произведения , как и в прошлый раз:

     Привет Картик,
    
    Спасибо, что написали доктору Мат.
    
    Учитывая идею вектора, для  естественно искать способ вычисления угла  t между векторами (a1, a2, a3) и (b1, b2, b3).2 - 2 || a || * || b || * cos [t]
    
    Упростите, и вы найдете это, где «x y» означает «x умножить на y»:
    
       a1 b1 + a2 b2 + a3 b3 = || a || * || b || * cos [t]
    
    Это дает понять, что левая сторона представляет собой полезную комбинацию a и b. 

    Это максимально краткий ответ, который можно было бы дать. Мы можем представить, что начнем с Закона косинусов в надежде найти угол. Найдя эту простую формулу, мы просто определяем левую часть как $$ \ mathbf {a} \ cdot \ mathbf {b} = a_1 b_1 + a_2 b_2 + a_3 b_3 $$, и мы доказали, что $$ \ mathbf { a} \ cdot \ mathbf {b} = || \ mathbf {a} || || \ mathbf {b} || \ cos (\ theta) $$

    (Кстати, обратите внимание, что для простоты набора мы часто использовали \ (| \ mathbf {u} | \) для величины, т.е.е. длина, но правильное обозначение — \ (|| \ mathbf {u} || \).)

    Теперь мы можем попробовать что-то подобное, чтобы изобрести перекрестное произведение :

     Для перекрестного произведения  , если у вас есть векторы a = (a1, a2, a3) и b = (b1, b2, b3), кажется очевидным, что вектор , перпендикулярный как a, так и b, будет полезный . Итак, ищем вектор (c1, c2, c3) такой, что
    
       a.c = 0
       b.c = 0
    
    Если вы решите эту систему двух уравнений относительно c1 и c2, вы найдете
    
       c1 = (a3 b2 - a2 b3) / (a2 b1 - a1 b2) * c3
       c2 = (-a3 b1 + a1 b3) / (a2 b1 - a1 b2) * c3
    
    Если вы выберете c3 как a2 b1 - a1 b2, это даст решение и немного уберет беспорядок.

    Определив скалярное произведение, мы используем его, чтобы определить, перпендикулярны ли два вектора. Полученная система уравнений выглядит так: $$ \ left \ {\ begin {matrix} a_1c_1 + a_2c_2 + a_3c_3 = 0 \\ b_1c_1 + b_2c_2 + b_3c_3 = 0 \ end {matrix} \ right. $$ Всего два уравнения для трех переменных, поэтому решение не единственное; как оказывается, он определяет только пропорции (т.е. направление результирующего вектора).

    Он рассматривает \ (c_3 \) как константу для решения (быстрый способ — использовать правило Крамера), замечая, что решение действительно выражает \ (c_1 \) и \ (c_2 \) как кратные \ (c_3 \), так что использование \ (c_3 \) в качестве знаменателя обеих дробей дает удобный результат: $$ \ mathbf {a} \ times \ mathbf {b} = \ mathbf {c} = (a_3b_2-a_2b_3, a_1b_3- a_3b_1, a_2b_1-a_1b_2) $$ В этом есть хороший шаблон, который можно переформулировать различными способами, в том числе с использованием определителей, которые являются лишь сокращенным способом написания именно таких выражений.

    Обратите внимание, что этот подход, как я предлагал ранее, фокусируется на создании перпендикулярного вектора; , мы еще даже не знаем, какое значение имеет его величина . Мы скоро вернемся к этому.

    Обратите внимание, что первоначальной целью было не найти скалярное произведение и векторное произведение! Требовалось найти угол , который ведет к скаляру, и перпендикулярный вектор . Когда они были обнаружены, было естественным назвать их скалярным произведением и векторным произведением и, в конечном итоге, использовать символы точки и перекрестного умножения для их различения.Фактическое развитие этих идей началось с другой точки зрения, поскольку сами векторы развивались лишь постепенно; изображения точки и креста появились примерно в 1880 году, после 40 лет развития.

    Можем ли мы доказать, что | u × v | = | u || v | sin

    θ?

    Мы получили определение перекрестного произведения как вектора, перпендикулярного данным векторам, но его величина была такой, какая возникла при простейшем выборе. Можем ли мы доказать, что величина равна площади параллелограмма? Я не нахожу этого ни в одном опубликованном ответе, поэтому я использую два, которые не были заархивированы.Во-первых, рассмотрим это из 2003 года:

    .
     Как можно доказать, что для двух векторов a и b векторное произведение (крестик b) = mod (a) .mod (b) .sin (theta) .n
    
    Откуда в уравнении входит синус - через определение детерминанта перекрестного произведения? 

    Доктор Фентон ответил:

     Привет, Питер!
    
    Я предполагаю, что n - это единичный вектор в направлении (a x b), поэтому ваш вопрос, по сути, почему
    
       | а х б | = | a || b | sin (θ),
    
    где θ - угол между a и b. Здесь | a | - модуль вектора a.Эта величина также является площадью параллелограмма со сторонами a и b.
    
            - - - - -
        а /:
         /: ч /
        /:
       / θ: /
       ----------
            б
    
    Высота h равна | a | sin (θ), поэтому площадь равна | a || b | sin (θ). 

    Тот факт, что величина — это площадь, не является частью доказательства, а является полезным фактом. 2 (θ) [а.2-2a_1a_2b_1b_2-2a_2a_3b_2b_3-2a_1a_3b_1b_3 $$

    Они равны!

    Использование векторной алгебры

    Самым сложным в конце была алгебра. С более широким пониманием алгебры векторов (доказуемым из чисто алгебраического определения, которое мы используем), можно, по крайней мере, сделать работу немного более аккуратной. Вот неархивированный вопрос 2006 года; Я собираюсь сильно сократить это обсуждение, удалив по пути ряд неправильных путей:

      Докажите, что || A x B || = || А ||.|| B || .sin θ, используя свойства векторного произведения. 
    
    Свойства:
    
    1. A x B = -B x A
    2. А х (В + С) = А х В + А х С
    3. A x A = 0
    4. r (A x B) = r (A) x B = A x (rB)
    5. A x (B x C) = (C.A) B - (A.B) C
    6. A. (B x C) = (A x B) .C
    7. B. (B x C) = (B x C) C = 0
    
    Понятия не имею, с чего начать! Мне дали подсказку «Используйте свойства, особенно 5, 6 и 7», но мне эти свойства кажутся бесполезными, потому что вопрос, который я пытаюсь доказать, не имеет отношения к свойствам. Меня очень, очень сбивает с толку тот факт, что я пытаюсь сделать что-то с величиной, но я должен манипулировать свойствами, которые не связаны с символами величины.2 = v.v для любого вектора v.
    
    Применение этого к этой проблеме дает вам скалярное произведение двух векторов, которые сами являются перекрестными произведениями. Свойство 6 позволит вам это переписать, а затем вам нужно будет использовать свойство 5 для дальнейшей перезаписи результата. Вам также потребуется использовать свойство скалярных произведений, которое
    
       A.B = || A || || B || соз (θ)
    
    где θ - угол между A и B. 

    Люси ответила (пропустив некоторые боковые следы):

     Привет, доктор Фентон!
    
    Я просмотрел ваши предложения и продвинулся намного дальше, но я еще не совсем понял.2 = (A x B). (A x B) 

    Доктор Фентон ответил:

     Привет Люси!
    
    Для удобства перепишу свойства:
    
    Свойства:
    
    1. A x B = -B x A
    2. А х (В + С) = А х В + А х С
    3. A x A = 0
    4. r (A x B) = r (A) x B = A x (rB)
    5. A x (B x C) = (C.A) B - (A.B) C
    6. A. (B x C) = (A x B) .C
    7. B. (B x C) = (B x C) C = 0
    
    Сначала примените свойство 6. Несколько сбивает с толку то, что A, B и C представляют как конкретные векторы (в вашей проблеме), так и общие векторы (в формулах). Позвольте мне переписать свойство 6 в терминах общих векторов U, V и W:
    
       U.(V x W) = (U x V) .W.
    
    Теперь примените его к (A x B). (A x B), где U = A x B, V = A и W = B:
    
       (A x B). (A x B) = ((A x B) x A) .B.
    
    Затем вы хотите применить свойство 5, чтобы упростить тройное векторное произведение с правой стороны, но для этого требуется, чтобы первое перекрестное произведение входило во второй фактор, поэтому вы должны сначала применить свойство 1, а затем вы можете применить свойство 5, получить
    
       - ((A.B) A - (A.A) B) .B.
    
    Теперь у вас есть вектор с разницей двух векторов. Точечные произведения A.B и A.A - это просто константы (скаляры): это похоже на
    
      - (sA - tB).2 (\ theta)) $ 

    $$ || A \ times B || = || A || || B || \ sin (\ theta) $$

    Для совсем другой точки зрения вы можете прочитать это из 2001 года, это вторая ссылка, которую я дал Авинашу:

     Перекрестные произведения; Вращение в трех измерениях 

    В следующий раз мы рассмотрим третью ссылку, длинное обсуждение точечных и перекрестных произведений и многое другое.

alexxlab

Добавить комментарий

Ваш адрес email не будет опубликован. Обязательные поля помечены *